SlideShare ist ein Scribd-Unternehmen logo
1 von 104
Downloaden Sie, um offline zu lesen
-S.R.Sruthi Meenaxshi
DYSLIPIDE
MIA
GUIDELIN
ES TO
PRACTICE
Dyslipidemia – Definition
• is, quite simply “abnormal lipid levels”, as
measured on a blood sample and which
reflects one of several disorders in the
metabolism of lipoproteins. It may be
classified as:
• hypercholesterolemia
• low levels of High Density Lipoproteins (HDL)
• hypertriglyceridemia
ATP III Classification
LDL cholesterol
<100 Optimal
100-129 Near optimal/above
optimal
130-159 Borderline high
160-189 high
>190 Very high
Total cholestrol
<200
200-239
>240
HDL cholestrol
>40
>60
Desirable
Borderline high
High
Low
high
lipid guidelines.pptx
lipid guidelines.pptx
lipid guidelines.pptx
lipid guidelines.pptx
lipid guidelines.pptx
lipid guidelines.pptx
-S.R.Sruthi Meenaxshi
DY
S
LIP
IDE
M
IA
G
U
IDE
LINE
STO
P
R
AC
TIC
E
lipid guidelines.pptx
Question 1
41 yo M evaluated in PCP appointment. He is
healthy with no symptoms, is sedentary and
obese. PMH and FHx non-contributory. He does
not smoke, drink, or use illicit drugs. He takes no
meds. On exam, BP 132/82, HR 80, and BMI 32.
Labs show total cholesterol 251, LDL 172, HDL
35, TG 220, HbA1c 5%.
Estimated 10 year risk for ASCVD is 3.4%
Which of the following is the most appropriate
management of this patient’s hyperlipidemia?
A: Ezetimibe
B: High-intensity rosuvastatin
C: Moderate intensity rosuvastatin
D. Niacin
E: Therapeutic Lifestyle modification
Question 2
80 yo M was hospitalized for a 5 day history of
acute leg ischemia treated with angioplasty and
stenting. Now asymptomatic. PMH significant
for CKD stage III and HTN on diltiazem, lisinopril,
ASA and plavix. Exam is normal.
Labs: AST 20, Total cholesterol 170, LDL 97, HDL
44, Cr. 1.8, TG 147 and GFR 35
Question 2
Which of the following is the most appropriate
therapy for secondary prevention of
cardiovascular disease in this patient?
A: High intensity rosuvastatin
B: Moderate intensity rousvastatin
C: Niacin
D: No additional Treatment
Question 3
48 yo M evaluated during follow up appointment. 3
months ago, he had a STEMI and underwent PCI
with bare metal stent of Left circumflex artery. He
was started on high intensity rosuvastatin at the
time. LFT’s normal and Cr level was normal. He is
now asymptomatic, no chest pain or muscle
pain.His meds are aspirin, metoprolol, lisinopril,
rosuvastatin, and plavix.
Exam and vitals normal. No muscle or abdominal
tenderness.
Question 3
Which of the following is the most appropriate
laboratory study to obtain at this visit?
A: Alanine aminotransferase level
B: Creatine Kinase Level
C: Fasting Lipid Panel
D. High sensitivity C-reactive protein level
Introduction
• Treatment of dyslipidemia is a cornerstone of
preventive cardiology, and reduction in low-
density lipoprotein (LDL-C) in select
populations reduces risk of atherosclerotic
cardiovascular disease (ASCVD) events in both
primary and secondary prevention.
Current Major Lipid Treatment
guidelines
1. 2014 American College of Cardiology/American Health Association
(ACC/AHA),4
2. 2014 U.S. Veterans Affairs/Department of Defense (VA-DoD).9
3. 2016 Canadian Cardiovascular Society (CCS),5
4. 2016 European Society of Cardiology/European Atherosclerosis
Society (ESC/EAS),6
5. 2016 US Preventive Services Task Force (USPSTF),7,8
6. 2017, American Association of Clinical Endocrinologist (AACE)
Examining differences between dyslipidemia guidelines can help
identify areas requiring further research and also potential
opportunities for harmonization of guidelines.
Lipid guidelines
• Previous ATP III (Adult Treatment Panel III)
looked at LDL goals in combination with
patient’s risk.
• Current ACC/AHA does not focus on pure lab
values, but on overall atherosclerotic
cardiovascular risk disease
Who should be tested ??
• USPSTF recommends screening:
– Men: >35yo
– Women: >45yo at increased increased risk
– Increased risk is defined as having: diabetes, CHD, family history of CHD in males <50yo and in
females <60yo, smokers, HTN, obesity
• All patients aged 40-79 years old should have their
10year risk for ASCVD using Pooled Cohort Equation
• Of note, European Federation of Clinical Chemistry and
Laboratory Medicine recommended in April 2016 non-
fasting measurements in most patients.
Risk Calculator
ASCVD risk <5% = LOW RISK
ASCVD 5- 7.5% =
INTERMEDIATE
ASCVD >7.5% = HIGH RISK
http://clincalc.com/Cardiology/ASCVD/PooledCohort.aspx
lipid guidelines.pptx
lipid guidelines.pptx
lipid guidelines.pptx
Who needs treatment?
• Group1: Clinical ASCVD
– ACS, history of MI, angina, stroke, TIA, PAD
• Group2: LDL >190
– Rule out secondary causes
• Group 3: All Diabetics without clinical ASCVD
– 40-75yo with LDL 70-189mg/dL
• Group 4: 10 year ASCVD risk >7.5%
– with LDL 70-189mg/dL
Management
• Start statin
– Select appropriate dose
for patient
– Keep potential side effects
and drug interactions in
mind
– If high/moderate intensity
not tolerate, use max
dose tolerated
• Intensity of Statin:
– Established ASCVD
• Age > 75 moderate intensity statin
• Age <75 high intensity statin
– LDL cholesterol >190mg/dL
• High intensity statin
– Diabetes 40-75yo w/LDL 70-189mg/dL and no ASCVD
• ASCVD 10 year risk >7.5% high intensity statin
• ASCVD 10 year risk < 7.5% moderate intensity statin
– No ASCVD or DM, but 10 year ASCVD > 7.5%
– Moderate to high intensity statin
– Adults >76 yo w/o history of ASCVD
• Insufficient evidence to recommend for/against statins
lipid guidelines.pptx
lipid guidelines.pptx
lipid guidelines.pptx
The risk of ASCVD and ASCVD-related mortality is substantially greater in
the presence of multiple risk factors. Since epidemiologic evidence indicates
that ASCVD risk factors frequently cluster, it should be expected that many
individuals have multiple risk factors.
Recommendations associated with this question:
What are the risk factors for ASCVD ?
R1. Identify risk factors that enable personalized and optimal therapy for
dyslipidemia. (Grade A; BEL 1).
Abbreviation: ASCVD, atherosclerotic cardiovascular disease.
Jellinger P, Handelsman Y, Rosenblit P, et al. Endocr Practice. 2017;23(4):479-497
Major Atherosclerotic Cardiovascular
Disease Risk Factors
Abbreviations: apo, apolipoprotein; ASCVD, atherosclerotic cardiovascular disease; HDL-C, high-density lipoprotein cholesterol;
hsCRP, highly sensitive C-reactive protein; LDL, low-density lipoprotein; LDL-C, low-density lipoprotein cholesterol; Lp-PLA2,
lipoprotein-associated phospholipase; PCOS, polycystic ovary syndrome.
AACE POSWC. Endocr Pract. 2005;11:126-134; ADA. Diabetes Care. 2017;40(Suppl 1):S1-S135; Brunzell JD, et al. Diabetes
Care.2008;31:811-822; Cromwell WC, et al. J Clin Lipidol. 2007;1:583-592;
MAJOR RISK FACTORS ADDITIONAL RISK FACTORS NON TRADITIONAL RISK
FACTORS
Advancing age
Increased Total Serum
cholesterol level
Increased Non HDL – C
Increased LDL – C
Low HDL –C
Diabetes Mellitus
Hypertension
Stage 3 or 4 chronic kidney
disease
Cigarette smoking
Family history of ASCVD
Obesity , abdominal
obesity
Family history of
hyperlipidemia
Increased small , dense
LDL-C
Increased APO – B
Increased LDL particle
concentration
Fasting / Postprandial
Hypertriglyceridemia
PCOS
Increased Lipoprotein (a)
Increased Clotting factors
Increased inflammation
markers (hs- CRP , LP-
PLA2)
Increased homocysteine
Apo E4 isoform
Increased uric acid
Increased Tgrich remnants
How can be the risk assessed?
R4. The 10-year risk of a coronary event (high, intermediate, or low)
should be determined by detailed assessment using one or more of the
following tools (Grade C; BEL 4, upgraded due to cost-effectiveness):
• Framingham Risk Assessment Tool
• MESA 10-year ASCVD Risk with Coronary Artery Calcification Calculator
• Reynolds Risk Score, which includes hsCRP and family history of premature ASCVD
• UKPDS risk engine to calculate ASCVD risk in individuals with T2DM
R7. When the HDL-C concentration is greater than 60 mg/dL, one risk
factor should be subtracted from an individual’s overall risk profile (Grade
B; BEL 2).
• R8. A classification of elevated TG should be incorporated into risk assessments to aid in
treatment decisions (Grade B; BEL 2).
Abbreviations: ASCVD, atherosclerotic cardiovascular disease; HDL-C, high-density lipoprotein cholesterol; hsCRP, high-sensitivity CRP;
MESA, Multi-Ethnic Study of Atherosclerosis; T2DM, type 2 diabetes mellitus; TG, triglycerides
Jellinger P, Handelsman Y, Rosenblit P, et al. Endocr Practice. 2017;23(4):479-497.
Framinghams risk assessment tool
MESA 10-year ASCVD Risk with
Coronary Artery Calcification
Calculator
Reynolds Risk Score, which includes
hsCRP and family history of premature
ASCVD
UKPDS risk engine to calculate ASCVD
risk in individuals with T2DM
ASCVD Risk Categories
• Low risk:
– No risk factors
• Moderate risk:
– 2 or fewer risk factors and a calculated 10-
year risk of less than 10%
• High risk:
– An ASCVD equivalent including diabetes
or stage 3 or 4 CKD with no other risk
factors, or individuals with 2 or more risk
factors and a 10-year risk of 10%-20%
• Very high risk:
– Established or recent hospitalization for
ACS; coronary, carotid or peripheral
vascular disease; diabetes or stage 3 or 4
CKD with 1 or more risk factors; a
calculated 10-year risk greater than 20%;
or HeFH
• Extreme risk:
– Progressive ASCVD, including unstable
angina that persists after achieving an
LDL-C less than 70 mg/dL, or established
clinical ASCVD with diabetes, stage 3 or 4
CKD, and/or HeFH, or in those with a
history of premature ASCVD (<55 years
of age for males or <65 years of age for
females)
– This category was added in this CPG based
on clinical trial evidence and supported by
meta-analyses that further lowering of
LDL-C produces better outcomes in
individuals with ACS. IMPROVE-IT
demonstrated lower rates of
cardiovascular events in those with ACS
when LDL-C levels were lowered to 53
mg/dL combining ezetimibe with statins.
Abbreviations: ACS, acute coronary syndrome; ASCVD, atherosclerotic cardiovascular disease; CKD, chronic kidney disease; CPG, clinical practice guideline; HeFH,
heterozygous familial hypercholesterolemia; LDL-C, low-density lipoprotein cholesterol; IMPROVE-IT, Improved Reduction of Outcomes: Vytorin Efficacy
International Trial.
AACE/ACE CPG. 2017;epub ahead of print; Cannon, CP, et al. N Engl J Med. 2015;372(25):2387-239; Jellinger P, Handelsman Y, Rosenblit P, et al. Endocr Practice.
2017;23(4):479-497.
Who should be screened and when ?
Familial Hypercholesterolemia
• R9. Individuals should be screened for FH when there is a family
history of:
• Premature ASCVD (definite MI or sudden death before age 55 years
in father or other male first-degree relative or before age 65 years
in mother or other female first-degree relative) or
• Elevated cholesterol levels (total, non-HDL, and/or LDL) consistent
with FH (Grade C; BEL 4, upgraded due to cost-effectiveness).
Adults With Diabetes
• R10. Annually screen all adult individuals with T1DM or T2DM for
dyslipidemia (Grade B; BEL 2).
Jellinger P, Handelsman Y, Rosenblit P, et al. Endocr Practice. 2017;23(4):479-
497
Who should be screened and when ?
Young Adults (Men Aged 20-45 Years, Women Aged 20-55 Years)
• R11. Evaluate all adults 20 years of age or older for dyslipidemia every 5
years as part of a global risk assessment (Grade C; BEL 4, upgraded due
to cost-effectiveness).
Middle-Aged Adults (Men Aged 45-65 Years, Women Aged 55-65
Years)
• R12. In the absence of ASCVD risk factors, screen middle-aged individuals for
dyslipidemia at least once every 1 to 2 years. More frequent lipid testing is
recommended when multiple global ASCVD risk factors are present (Grade A; BEL 1).
• R13. The frequency of lipid testing should be based on individual clinical circumstances
and the clinician’s best judgment (Grade C; BEL 4, upgraded due to cost-effectiveness).
Jellinger P, Handelsman Y, Rosenblit P, et al. Endocr Practice.
2017;23(4):479-497
• R14. Annually screen older adults with 0 to 1
ASCVD risk factor for dyslipidemia (Grade A; BEL
1).
• R15. Older adults should undergo lipid
assessment if they have multiple ASCVD global
risk factors (i.e., other than age) (Grade C; BEL 4,
upgraded due to cost-effectiveness).
• R16. Screening for this group is based on age and
risk, but not gender; therefore, older women
should be screened in the same way as older men
(Grade A; BEL 1).
Older Adults (Older Than 65 Years)
Whom to screen ?
*Men younger than 55 years and women younger than 65 years of age in first-degree relative. BMI, body
mass index.
How to screen ?
ApoB, apolipoprotein B; eGFR, estimated glomerular filtration rate; HDL-C,high-density lipoprotein cholesterol; LDL-C, low-
density lipoprotein cholesterol; TC, total cholesterol; TG, triglycerides
Familial Hypercholesterolemia:
Diagnosis
• FH diagnostic criteria include lipid levels and family history, physical
symptoms (if any), and genetic analysis (if available)1
• Three clinical diagnostic tools:2-3
• Simon Broome Register Diagnostic Criteria
• Dutch Lipid Clinic Network Diagnostic Criteria
• U.S. MEDPED
• Factors that lead to an FH diagnosis include:
• Premature ASCVD, fasting LDL-C >190 mg/dL, the presence of tendon
xanthomas, full corneal arcus in individuals <40 years of age, or a family
history of high cholesterol and/or premature ASCVD1
• While genetic testing may identify FH, it is not commonly used in the
United States due to cost and lack of payer coverage1
Abbreviations: ASCVD, atherosclerotic cardiovascular disease; FH, familial hypercholesterolemia; LDL-C, low-density lipoprotein
cholesterol; MEDPED, Make Early Diagnoses Prevent Early Deaths Program Diagnostic Criteria.
1. Bouhairie VE, et al. Cardiol Clin. 2015;33:169-179; 2. Haralambos K, et al. Curr Opin Lipidol. 2016;27:367-374; 3. Turgeon RD, et al.
Can Fam Physician. 2016;62:32-37.
Familial Hypercholesterolemia:
Prevalence and Risk
• FH is caused by genetic mutations passed on by:
• One parent (heterozygous, HeFH)1
• Both parents (homozygous, HoFH)1
• HoFH prevalence ranges from 1 in 160,000 to 1 in 250,0002,3
• Individuals with HoFH have extremely high LDL-C levels (>500 mg/dL) and premature CV risk4
• Many with HoFH experience their first coronary event in childhood or adolescence4
• HeFH prevalence ranges from 1 in 200 to 1 in 2503
• Individuals with HeFH can present with LDL-C levels 90 to 500
mg/dL and have premature CV risk4
• On average, individuals with HeFH experience their first
coronary event at age 42 (about 20 years younger than the
general population)4
• Early treatment is recommended for all individuals with FH, with a
goal of reducing LDL-C levels by 50% from baseline3
Abbreviations: CV, cerebrovascular; FH, familial hypercholesterolemia; HeFH, heterozygous familial hypercholesterolemia; HoFH, homozygous familial
hypercholesterolemia; LDL-C, low-density lipoprotein cholesterol.
1. Zimmerman MP. Am Health Drug Benefits. 2015;8:436-442; 2. Goldstein J, et al. The Metabolic and Molecular Bases of Inherited Disease. 7th ed. New
York, NY: McGraw-Hill; 1995: 1981-2030; 3. Bouhairie VE, et al. Cardiol Clin. 2015;33:169-179; 4. Turgeon RD, et al. Can Fam Physician. 2016;62:32-37.
Which screening tests should be
used?
Fasting Lipid Profile
• R19. Use a fasting lipid profile to ensure the most precise lipid assessment; this should
include total cholesterol, LDL-C, TG, and non-HDL-C (Grade C; BEL 4, upgraded due to cost-
effectiveness).
• R20. Lipids, including TG, can be measured in the non-fasting state if fasting determinations
are impractical (Grade D).
LDL-C
• R21. LDL-C may be estimated using the Friedewald equation: LDL-C = (total cholesterol –
HDL-C) – TG/5; however, this method is valid only for values obtained during the fasting state
and becomes increasingly inaccurate when TG levels are greater than 200 mg/dL, and
becomes invalid when TG levels are greater than 400 mg/dL (Grade C; BEL 3).
• R22. LDL-C should be directly measured in certain high-risk individuals, such as those with
fasting TG levels greater than 250 mg/dL or those with diabetes or known vascular disease
(Grade C; BEL 3).
HDL-C
• R23. Measurement of HDL-C should be included in screening tests for
dyslipidemia (Grade B; BEL 2).
Abbreviations: HDL-C, high-density lipoprotein cholesterol; LDL-C, low-density lipoprotein cholesterol; TG, triglycerides.
Jellinger P, Handelsman Y, Rosenblit P, et al. Endocr Practice. 2017;23(4):479-497.
Which screening tests should be
used?
Non-HDL-C
• R24. Non-HDL-C (total cholesterol minus HDL-C) should be calculated to assist risk
stratification in individuals with moderately elevated TG (200 to 500 mg/dL), diabetes,
and/or established ASCVD (Grade B; BEL 2).
• R25. If insulin resistance is suspected, non-HDL-C should be evaluated to gain useful
information regarding the individual’s total atherogenic lipoprotein burden (Grade D).
Triglycerides
• R26. TG levels should be part of routine lipid screening: moderate elevations (≥150 mg/dL)
may identify individuals at risk for insulin resistance syndrome and levels ≥200 mg/dL may
identify individuals at substantially increased ASCVD risk (Grade B; BEL 2).
Apolipoproteins
• R27. Apo B and/or an apo B/apo A1 ratio calculation and evaluation may be useful in at-risk
individuals (TG ≥150, HDL-C <40, prior ASCVD event, T2DM, and/or insulin resistance
syndrome [even at target LDL-C levels]) to assess residual risk and guide decision-making
(Grade A; BEL 1).
• R28. Apo B measurements (reflecting the particle concentration of LDL and all
other atherogenic lipoproteins) may be useful to assess the success of
LDL-C–lowering therapy (Grade A; BEL 1).
Abbreviations: apo, apolipoprotein; ASCVD, atherosclerotic cardiovascular disease; HDL-C, high-density lipoprotein cholesterol; LDL-C, low-
density lipoprotein cholesterol; T2DM, type 2 diabetes mellitus; TG, triglycerides.
Jellinger P, Handelsman Y, Rosenblit P, et al. Endocr Practice. 2017;23(4):479-497
Additional screening test
Coronary artery
calcification
• R33. Coronary
artery
calcification
measurement
has been shown
to be of high
predictive value
and is useful in
refining risk
stratification to
determine the
need for more
aggressive
treatment
strategies (Grade
B; BEL 2).
hsCRP
• R30. Use hsCRP
to stratify ASCVD
risk in individuals
with a standard
risk assessment
that is borderline,
or in those with
an intermediate
or higher risk
with an LDL-C
concentration
less than 130
mg/dL (Grade B;
BEL 2).
Lp-PLA2
• R31. Measure
lipoprotein-
associated
phospholipase A2
(Lp-PLA2), which
in some studies
has
demonstrated
more specificity
than hsCRP, when
it is necessary to
further stratify an
individual’s
ASCVD risk,
especially in the
presence of
hsCRP elevations
(Grade A; BEL 1).
Homocysteine
• R32. The routine
measurement of
homocysteine,
uric acid,
plasminogen
activator
inhibitor-1, or
other
inflammatory
markers is not
recommended
because the
benefit of doing
so is not
sufficiently
proven (Grade
D).
Carotid intima
media thickness
• R34. Carotid
intima media
thickness may be
considered to
refine risk
stratification to
determine
the need for
more aggressive
ASCVD
preventive
strategies (Grade
B; BEL 2).
Abbreviations: apo, apolipoprotein; ASCVD, atherosclerotic cardiovascular disease; HDL-C, high-density lipoprotein cholesterol; LDL-C,
low-density lipoprotein cholesterol; T2DM, type 2 diabetes mellitus; TG, triglycerides.
Jellinger P, Handelsman Y, Rosenblit P, et al. Endocr Practice. 2017;23(4):479-497
•R5. Special attention should be given to assessing women for ASCVD risk by determining the
10-year risk (high, intermediate, or low) of a coronary event using the Reynolds Risk Score or
the Framingham (Grade C; BEL 4, upgraded due to cost-effectiveness).
Risk Assessment
•R72. Women should be evaluated for their ASCVD risk and be treated with pharmacotherapy if
lifestyle intervention is insufficient (Grade C; BEL 4; upgraded due to potential benefit).
•R73. Hormone replacement therapy for the treatment of dyslipidemia in postmenopausal
women is not recommended (Grade A; BEL 1).
•An HDL-C concentration <40 mg/dL is an established independent risk factor for ASCVD in both
men and women. However, because HDL-C levels tend to be higher in women than in men, an
HDL-C concentration <50 mg/dL in women is also considered a marginal risk factor.
•In stark contrast to findings in men, very low HDL-C (<40 mg/dL) is an independent risk factor
for ASCVD development and mortality in women, even in the presence of total cholesterol
concentrations less than 200 mg/dL or normal LDL-C and/or TG levels. Compared with women
with high HDL-C, women with low HDL-C have a nearly 3-fold elevated risk of ASCVD.
Treatment options
What special consideration should be given for women ?
Jellinger P, Handelsman Y, Rosenblit P, et al. Endocr Practice. 2017;23(4):479-497
Common Secondary Causes of
Dyslipidemia
What are lipid treatment goals?
R35. Treatment goals for dyslipidemia should be personalized according to
levels of risk (Grade A; BEL 1).
R36. For individuals at low risk (i.e., with no risk factors), an LDL-C goal of
less than 130 mg/dL is recommended (Grade A; BEL 1).
R37. For individuals at moderate risk (i.e., with 2 or fewer risk factors and a
calculated 10-year risk of less than 10%), an LDL-C goal of less than 100
mg/dL is recommended (Grade A; BEL 1).
R38. For individuals at high risk (i.e., with an ASCVD equivalent including diabetes or
stage 3 or 4 CKD with no other risk factors, or individuals with 2 or more risk factors
and a 10-year risk of 10%-20%), an LDL-C goal of less than 100 mg/dL is
recommended (Grade A; BEL 1).
Jellinger P, Handelsman Y, Rosenblit P, et al. Endocr Practice. 2017;23(4):479-497.
R39. For individuals at very high risk (i.e., with established or recent
hospitalization for ACS; coronary, carotid or peripheral vascular disease; diabetes
or stage 3 or 4 CKD with 1 or more risk factors; a calculated 10-year risk greater
than 20%; or HeFH), an LDL-C goal of less than 70 mg/dL is recommended (Grade
A; BEL 1).
R40. For individuals at extreme risk (i.e., with progressive ASCVD, including unstable
angina that persists after achieving an LDL-C less than 70 mg/dL, or established clinical
ASCVD in individuals with diabetes, stage 3 or 4 CKD, and/or HeFH, or in individuals
with a history of premature ASCVD (<55 years of age for males or <65 years of age for
females), an LDL-C goal of less than 55 mg/dL is recommended (Grade A; BEL 1).
R41. An LDL-C goal of <100 mg/dL is considered “acceptable” for children and
adolescents, with 100 to 129 mg/dL considered “borderline” and 130 mg/dL or
greater considered “high” (based on recommendations from the American
Academy of Pediatrics) (Grade D).
Jellinger P, Handelsman Y, Rosenblit P, et al. Endocr Practice. 2017;23(4):479-497.
High-Density Lipoprotein Cholesterol
•R42. HDL-C should be greater than 40 mg/dL, but also as high as possible, primarily through the use of
lifestyle interventions (e.g., weight loss, physical activity, and tobacco cessation), and if risk factors are
present (e.g., borderline elevated LDL-C levels, a family history of premature ASCVD, or a personal
history of ASCVD), also through the use of pharmacotherapy primarily focused on reducing LDL-C
(Grade A; BEL 1).
Non–High-Density Lipoprotein Cholesterol
•R43. For most individuals, a non–HDL-C goal (total cholesterol minus HDL-C) 30 mg/dL higher than the
individual’s specific LDL-C goal is recommended (Grade D).
•R44. For individuals at extreme risk, a non-HDL-C goal 25 mg/dL higher than the individual-specific
LDL-C goal is recommended (Grade A; BEL 1).
Apolipoproteins
•R45. For individuals at increased risk of ASCVD, including those with diabetes, an optimal apo B goal is
less than 90 mg/dL, while for individuals with established ASCVD or diabetes plus 1 or more additional
risk factor(s), an optimal apo B goal is less than 80 mg/dL, and for individuals at extreme risk, an
optimal apo B goal is less than 70 mg/dL (Grade A; BEL 1).
Triglycerides
•R46. TG goals of less than 150mg/dL are recommended (Grade A; BEL 1).
Treatment goals
AACE/ACE 2017;epub ahead of print; Baigent C, et al. Lancet. 2010;376:1670-1681; Boekholdt SM, et al. J Am Coll Cardiol. 2014;64(5):485-494;
Brunzell JD, et al. Diabetes Care. 2008;31:811-822; Cannon CP, et al. N Engl J Med. 2015;372(25):2387-2397; Heart Protection Study
Collaborative
Group. Lancet. 2002;360:7-22; Jellinger P, Handelsman Y, Rosenblit P, et al. Endocr Practice. 2017;23(4):479-497; Ridker PM, J Am Coll Cardiol.
2005;45:1644-1648; Sever PS, et al. Lancet. 2003;361:1149-1158; Shepherd J, et al. Lancet. 2002;360:1623-1630; Weiner DE, et al. J Am Soc
Nephrol. 2004;15(5):1307-1315.
TREATMENT GOALS
Barter PJ, et al. J Intern Med. 2006;259:247-258; Boekholdt SM, et al. J Am Coll Cardiol. 2014;64(5):485-494; Brunzell JD, et al. Diabetes Care. 2008;31:811-822;
Cannon CP, et al. N Engl J Med. 2015;372(25):2387-2397; Grundy SM, et al. Circulation. 2004;110:227-239; Heart Protection Study Collaborative Group. Lancet.
2002;360:7-22; Jellinger P, Handelsman Y, Rosenblit P, et al. Endocr Practice. 2017;23(4):479-497; Lloyd-Jones DM, et al. Am J Cardiol. 2004;94:20-24; McClelland RL,
et al. J Am Coll Cardiol. 2015;66(15):1643-1653; NHLBI. NIH Publication No. 02-5215. 2002; Ridker PM, J Am Coll Cardiol. 2005;45:1644-1648; Ridker PM, et al. JAMA.
2007;297(6):611-619; Sever PS, et al. Lancet. 2003;361:1149-1158; Shepherd J, et al. Lancet. 2002;360:1623-1630; Smith SC Jr, et al. Circulation. 2006;113:2363-2372;
Stevens RJ, et al. Clin Sci. 2001;101(6):671-679; Stone NJ. Am J Med. 1996;101:4A40S-48S; Weiner DE, et al. J Am Soc Nephrol. 2004;15(5):1307-1315.
CLASSIFICATION OF ELEVATED
TRIGLYCERIDE LEVELS
Einhorn D, et al. Endocr Pract. 2003;9:237-252; Frick MH, et al. NEJM. 1987;317:1237-1245; Jellinger P,
Handelsman Y, Rosenblit P, et al. Endocr Practice. 2017;23(4):479-497; Keech A, et al. Lancet. 2005;366:1849-
1861; NHLBI. NIH Publication No. 02-5215. 2002; Tenaknen L, et al. Arch Intern Med. 2006;166:743-748.
TG levels that are even moderately elevated (≥150 mg/dL) may identify
individuals at risk for the insulin resistance syndrome. TG levels ≥200 mg/dL
may indicate a substantial increase in ASCVD risk. Hypertriglyceridemia is
also commonly associated with a procoagulant state and hypertension.
What treatments are available for
dyslipidemia?
Treatment categories for dyslipidemia:
– Lifestyle changes
• Physical activity
• Medical nutrition therapy
• Smoking cessation
– Pharmacologic therapy
• Statins
• Fibrates
• Omega-3 fish oil
• Niacin
• Bile acid sequestrants
• Cholesterol absorption inhibitors
• PCSK9 inhibitors
• MTP inhibitor
• Antisense apo B oligonucleotide
• Combination therapies
Abbreviations: apo, apolipoprotein; MTP, microsomal transfer protein;
PCSK9, proprotein convertase subtilisin/kexin type 9.
Non pharmacological lipid treatment
Physical Activity
• R48. A reasonable and feasible approach to fitness therapy (i.e., exercise programs that include at least 30 minutes of
moderate-intensity physical activity [consuming 4-7 kcal/min] 4 to 6 times weekly, with an expenditure of at least 200
kcal/day) is recommended; suggested activities include brisk walking, riding a stationary bike, water aerobics,
cleaning/scrubbing, mowing the lawn, and sporting activities (Grade A; BEL 1).
• R49. Daily physical activity goals can be met in a single session or in multiple sessions throughout the course of a day (10
minutes minimum per session); for some individuals, breaking activity up throughout the day may help improve adherence
with physical activity programs (Grade A; BEL 1).
• R50. In addition to aerobic activity, muscle-strengthening activity is recommended at least 2 days a week (Grade A; BEL 1).
Medical Nutrition Therapy
• R51. For adults, a reduced-calorie diet consisting of fruits and vegetables (combined ≥5 servings/day), grains (primarily whole
grains), fish, and lean meats is recommended (Grade A; BEL 1).
• R52. For adults, the intake of saturated fats, trans-fats, and cholesterol should be limited, while LDL-C-lowering
macronutrient intake should include plant stanols/sterols (~2 g/ day) and soluble fiber (10-25 g/day) (Grade A; BEL 1).
• R53. Primary preventive nutrition consisting of healthy lifestyle habits is recommended in all healthy children (Grade A; BEL
1).
Smoking Cessation
• R54. Tobacco cessation should be strongly encouraged and facilitated (Grade A; BEL 2; upgraded due to potential benefit).
Jellinger P, Handelsman Y, Rosenblit P, et al. Endocr Practice. 2017;23(4):479-497
Statins, Fibrates
Statins
• R56. Statin therapy is recommended as the primary pharmacologic agent to achieve target LDL-C goals on the basis
of morbidity and mortality outcome trials (Grade A; BEL 1).
• R57. For clinical decision making, mild elevations in blood glucose levels and/or an increased risk of new-onset
T2DM associated with intensive statin therapy do not outweigh the benefits of statin therapy for ASCVD risk
reduction (Grade A, BEL 1).
• R58. In individuals within high-risk and very high-risk categories, further lowering of LDL-C beyond established
targets with statins results in additional ASCVD event reduction and may be considered (Grade A, BEL 1).
• R59. Very high-risk individuals with established coronary, carotid, and peripheral vascular disease, or diabetes, who
also have at least 1 additional risk factor, should be treated with statins to target a reduced LDL-C treatment goal of
<70 mg/dL (Grade A, BEL 1).
• R60. Extreme risk individuals should be treated with statins or with combination therapy to target an even lower
LDL-C treatment goal of <55 mg/dL (Grade A, BEL 1).
Fibrates
• R61. Fibrates should be used to treat severe hypertriglyceridemia (TG >500 mg/dL) (Grade A; BEL 1).
• R62. Fibrates may improve ASCVD outcomes in primary and secondary prevention when
TG concentrations are 200 mg/dL and HDL-C concentrations <40 mg/dL (Grade A; BEL 1).
Jellinger P, Handelsman Y, Rosenblit P, et al. Endocr Practice. 2017;23(4):479-497
Statin
Main Considerations
• Liver function test prior to therapy and as clinically indicated thereafter
• Myalgias and muscle weakness in some individuals
• Potential for drug-drug interaction between some statins and CYP450 3A4 inhibitors,
cyclosporine, warfarin, and protease inhibitors
• Myopathy/rhabdomyolysis in rare cases; increased risk with coadministration of some drugs
(see product labeling)
• Simvastatin dosages should not exceed 40 mg in most individuals; dosages of 80 mg are no
longer recommended except in those who have tolerated 80 mg for 12 months or more
without muscle toxicity
• Do not exceed 20 mg simvastatin daily with amlodipine or ranolazine
• Plasma elevations of rosuvastatin may be higher among Asian persons than other ethnic
groups
• New-onset diabetes is increased in individuals treated with statins; however, it is dose-
related, occurs primarily in individuals with MetS, appears to be less common with
pravastatin and possibly pitavastatin, and occurs overall to a lesser extent than the associated
decrease in ASCVD
Metabolic Effects
• Primarily ↓ LDL-C 21%-55% by competitively inhibiting rate-limiting step of cholesterol
synthesis in the liver, leading to upregulation of hepatic LDL receptors
• Effects on TG and HDL-C are less pronounced (↓ TG 6%-30% and ↑ HDL-C 2%-10%)
Bissonnette S, et al. Can J Cardiol. 2006;22:1035-1044; Denke M, et al. Diab Vasc Dis Res. 2006;3:93-102; Jellinger P,
Handelsman Y, Rosenblit P, et al. Endocr Practice. 2017;23(4):479-497; Preiss D, et al. JAMA. 2011;305: 2556-2564
Statin and dosages
Agent
Usual recommended
starting daily dosage
Dosage range
Method of
administration
Statins
Lovastatin 20 mg 10-80 mg Oral
Pravastatin 40 mg 10-80 mg Oral
Simvastatin 20-40 mg 5-80 mga Oral
Fluvastatin 40 mg 20-80 mg Oral
Atorvastatin 10-20 mg 10-80 mg Oral
Rosuvastatin 10 mg 5-40 mg Oral
Pitavastatin 2 mg 2-4 mg Oral
Simvastatin, 80 mg, not approved for therapy unless individual has been on
treatment for more than 1 year without myopathy.
Crestor (rosuvastatin calcium); [PI]; 2016; Jellinger P, Handelsman Y, Rosenblit P, et al. Endocr Practice. 2017;23(4):479-497; Lescol
(fluvastatin sodium) [PI]; 2012 Lipitor (atorvastatin calcium) [PI]; 2015; Livalo (pitavastatin) [PI]; 2013; ; Mevacor (lovastatin) [PI];
2014; Pravachol (pravastatin sodium) [PI]; 2016; Zocor (simvastatin) [PI]; 2015.
Fibrates
Metabolic Effects:
• Primarily ↓ TG 20%-35%, ↑ HDL-C 6%-18% by stimulating lipoprotein lipase activity
• Fenofibrate may ↓ TC and LDL-C 20%-25%
• Lower VLDL-C and LDL-C; reciprocal rise in LDL-C transforms the profile into a less
atherogenic form by shifting fewer LDL particles to larger size
• Fenofibrate ↓ fibrinogen level
Abbreviations: HDL-C, high-density lipoprotein cholesterol; LDL, low-density lipoprotein, LDL-C, low-density lipoprotein cholesterol; TC, total
cholesterol; TG, triglycerides; VLDL-C, very low-density lipoprotein cholesterol.
Aguilar-Salinas CA, et al. Metabolism. 2001;50:729-733; Athyros VG, et al. Coron Artery Dis. 1995;6:25-1256; Avellone G, et al. Blood Coagul
Fibrinolysis. 1995;6:543-548; Bröijersen A, et al. Arterioscler Thromb Vasc Biol. 1996;16:511-516; Bröijersén A, et al. Thromb Haemost.
1996;76:171-176; Davidson MH, et al. Am J Cardiol. 2007;99:3C-18C; Farnier M, et al. Eur Heart J. 2005;26:897-905; Guyton JR, et al. Arch
Intern Med. 2000;160:1177-1184; Hottelart C, et al. Nephron. 2002;92:536-541; Insua A, et al. Endocr Pract. 2002;8:96-101; Jellinger P,
Handelsman Y, Rosenblit P, et al. Endocr Practice. 2017;23(4):479-497; Kockx M, et al. Thromb Haemost. 1997;78:1167-1172; Lopid
(gemfibrozil) [PI] 2010; McKenney JM, et al. J Am Coll Cardiol. 2006;47:1584-1587; Syvänne M, et al. Atherosclerosis. 2004;172:267-272;
Tricor (fenofibrate) [PI]; 2010; Trilipix (fenofibric acid) [PI]; 2016; Westphal S, et al. Lancet. 2001; 358:39-40.
Fibrates metabolic effects and main
consideration
Main Considerations:
Gemfibrozil may ↑ LDL-C 10%-15%
GI symptoms, possible cholelithiasis
May potentiate effects of orally administered anticoagulants
Gemfibrozil may ↑ fibrinogen level
Gemfibrozil and fenofibrate can ↑ homocysteine independent of vitamin
concentrations
May cause muscle disorders; myopathy/rhabdomyolysis when used with
statin
Fibrates are associated with increased serum creatinine levels, which may
not reflect renal dysfunction
Fenofibrate dose should be cut by two-thirds and gemofibrozil by one-half
when eGFR is 15-60, and fibrates should be avoided when eGFR is <15
Can improve diabetic retinopathy
Omega 3 fish oil
Metabolic Effects:
• ↓ TG 27%-45%, TC 7%-10%, VLDL-C 20%-42%, apo B 4%, and non-HDL-C 8%-14%
in individuals with severe hypertriglyceridemia most likely by reducing hepatic
VLDL-TG synthesis and/or secretion and enhancing TG clearance from circulating
VLDL particles. Other potential mechanisms of action include: increased ß-
oxidation; inhibition of acyl-CoA; 1,2-diacylglyceral acyltransferase; decreased
hepatic lipogenesis; and increased plasma lipoprotein activity.
• Icosapent ethyl ↓ LDL-C 5%, whereas, omega-3-acid ethyl esters ↑ LDL-C 45%
Omega 3 fatty aids
Assess TG levels prior to initiating and periodically during therapy.
Omega-3-acid ethyl esters can increase LDL-C levels. Monitor LDL-C levels during
treatment.
May prolong bleeding time. Monitor coagulation status periodically in patients receiving
treatment with omega-3 fatty acids and other drugs affecting coagulation.
Monitor ALT and AST levels periodically during treatment in patients with hepatic
impairment. Some patients may experience increases in ALT levels only.
Exercise caution when treating patients with a known hypersensitivity to fish and/or
shellfish.
Omega 3 fattyacids
Assess TG levels prior to initiating and periodically during therapy.
Omega-3-acid ethyl esters can increase LDL-C levels. Monitor LDL-C levels during
treatment.
May prolong bleeding time. Monitor coagulation status periodically in patients
receiving treatment with omega-3 fatty acids and other drugs affecting coagulation.
Monitor ALT and AST levels periodically during treatment in patients with hepatic
impairment. Some patients may experience increases in ALT levels only.
Exercise caution when treating patients with a known hypersensitivity to fish and/or
shellfish.
Agent
Usual recommended
starting daily dosage
Dosage range
Method of
administration
Bile acid sequestrants
Cholestyramine 8-16 g 4-24 g Oral
Colestipol 2 g 2-16 g Oral
Colesevelam 3.8 g 3.8-4.5 g Oral
Metabolic Effects:
• Primarily ↓ LDL-C 15%-25% by binding bile acids and preventing their
reabsorption in the ileum (causing hepatic cholesterol depletion and LDL-
receptor upregulation)
• Colesevelam ↓ glucose and hemoglobin A1C (~0.5%); FDA-approved to
treat T2DM
Abbreviations: A1C, glycated hemoglobin; FDA, Food and Drug Administration; LDL, low-density lipoprotein; LDL-C, low-
density lipoprotein cholesterol; T2DM, type 2 diabetes mellitus; TG, triglyceride.
Colestid (colestipol hydrochloride) [PI]; 2014; Jellinger P, Handelsman Y, Rosenblit P, et al. Endocr Practice. 2017;23(4):479-
497; Prevalite (cholestyramine for oral suspension, USP) [PI]; 2015; WelChol (colesevelam hydrochloride) [PI]; 2014; Zieve FJ,
et al. Ther. 2007;29:74-839:74-83.
Bile acid sequestrants
• Main Considerations:
•May ↑ serum TG
•Frequent constipation and/or bloating, which
can reduce adherence
•Many potential drug interactions (decreased
drug absorption), less so with colesevelam (see
product labeling)
•May reduce absorption of folic acid and fat-
soluble vitamins such as vitamins A, D, and K
Metabolic Effects
• Primarily ↓ LDL-C 10%-18% by inhibiting intestinal absorption of cholesterol and
decreasing delivery to the liver, leading to upregulation of hepatic LDL receptors
• ↓ Apo B 11%-16%
• In combination with statins, additional ↓ LDL-C 25%, total ↓ LDL-C 34%-61%
• In combination with fenofibrate, ↓ LDL-C 20%-22% and ↓ apo B 25%-26% without
reducing ↑ HDL-C
Main Considerations
• Myopathy/rhabdomyolysis (rare)
• When coadministered with statins or fenofibrate, risks associated with those
drugs remain (e.g., myopathy/ rhabdomyolysis, cholelithiasis)
Niacin
• Metabolic Effects:
• ↓ LDL-C 10%-25%, ↓ TG 20%-30%, ↑ HDL-C 10%-35%
by decreasing hepatic synthesis of LDL-C and VLDL-C
• ↓ Lipoprotein (a)
• Transforms LDL-C to less atherogenic form by increasing
average particle size and also decreases LDL particle
concentration
Usual recommended
starting daily dosage
Dosage range
Method of
administration
Niacin (nicotinic acid)
Immediate release 250 mg 250-3,000 mg Oral
Extended release 500 mg 500-2,000 mg Oral
Niacin
• Main Considerations:
• Potential for frequent skin flushing, pruritus,
abdominal discomfort, hepatoxicity (rare but
may be severe), nausea, peptic ulcer, atrial
fibrillation
• Deleterious effect on serum glucose at higher
dosages
• Increases uric acid levels; may lead to gout
Metabolic Effects:
• ↓LDL-C 48%-71%, ↓ non-HDL-C 49%-58%, ↓TC 36%-42%,
↓Apo B 42%-55% by inhibiting PCSK9 binding with LDLRs,
increasing the number of LDLRs available to clear LDL, and
lowering LDL-C levels
Jellinger P, Handelsman Y, Rosenblit P, et al. Endocr Practice. 2017;23(4):479-
497; Praluent (alirocumab) [PI] 2015; Repatha (evolocumab) [PI]; 2016.
• Main Considerations:
• Require subcutaneous self-injection; refrigeration generally needed
• Overall levels of adverse reactions and discontinuation very low
• Adverse reactions with significantly different rates between drug and
placebo were: local injection site reactions and influenza
• The most common adverse reactions with similar rates for drug vs. placebo
were:
• Alirocumab: nasopharyngitis, influenza, urinary tract infections,
diarrhea, bronchitis, and myalgia
• Evolocumab: nasopharyngitis, back pain,
and upper respiratory tract infection
MTP Inhibitor
Metabolic Effects:
• ↓ Up to LDL-C 40%, TC 36%, apo B 39%, TG 45%, and non-HDL-C 40% (depending
on dose) in individuals with HoFH by binding and inhibiting MTP, which inhibits
synthesis of chylomicrons and VLDL
Abbreviations: ALT, aspartate amino transferase; AST, amino alanine transferase; FDA, Food dministration; HDL-C, high-density lipoprotein
cholesterol; HoFH, homozygous familial hypercholesterolemia; LDL-C, low-density lipoprotein cholesterol; MTP, microsomal transfer protein; REMS,
Risk Evaluation and Mitigation Strategy; TG, triglycerides; VLDL, very low-density lipoprotein.
Jellinger P, Handelsman Y, Rosenblit P, et al. Endocr Practice. 2017;23(4):479-497; Juxtapid (lomitapide) [PI]; 2012.
MTP Inhibitor
• Main Considerations:
• Can cause increases in transminases (ALT, AST); monitoring of ALT,
AST, alkaline phosphatase, and total bilirubin prior to initiation, and
of ALT and AST during treatment, is required per FDA REMS
• Causes increases in hepatic fat (steatosis) with or without
concomitant elevated transminases, which may be a risk for
progressive liver diseases
• Also causes steatosis of the small intestine with resulting abdominal
pain and steatorrhea unless a very-low-fat diet is followed; may also
cause fat-soluble vitamin deficiency unless vitamin supplements are
taken
• Caution should be exercised when used with other drugs with
potential hepatoxicity; because of hepatoxicity risk, only available
through REMS program
• Metabolic Effects
• ↓ LDL-C 21%, TC 19%, apo B 24%, and non-HDL-C
22% in individuals with HoFH by degrading mRNA
for apo B-100, the principal apolipoprotein
needed for hepatic synthesis of VLDL (and
subsequent intra-plasma production of IDL and
LDL)
Agent
Usual recommended
starting daily dosage
Dosage range
Method of
administration
Anti-sense apolipoprotein B oligonucleotide
Mipomersen
(SQ injection)
200 mg once weekly 200 mg once weekly SQ
mipomersen
• Main Considerations:
• Can cause increases in transminases (ALT,
AST); monitoring of ALT, AST, alkaline phosphatase, and
total bilirubin before initiation, and of ALT and AST
during treatment is recommended
• Causes increases in hepatic fat (steatosis) with or
without concomitant elevated transminases, which may
be a risk for progressive liver diseases
• Caution should be exercised when used with other drugs
with potential hepatoxicity; because of hepatoxicity risk,
only available through REMS program
Jellinger P, Handelsman Y, Rosenblit P, et al. Endocr Practice. 2017;23(4):479-497; Kynamro
(mipomersen sodium) Injection [PI]; 2016
How should treatment be monitored?
• R75. Reassess individuals’ lipid status 6 weeks after therapy initiation and
again at 6-week intervals until the treatment goal is achieved (Grade D;
BEL 4).
R76. While on stable lipid therapy, individuals should be tested at 6- to 12-month
intervals (Grade D; BEL 4).
R77. While on stable lipid therapy, the specific interval of testing should depend on
individual adherence to therapy and lipid profile consistency; if adherence is a
concern or the lipid profile is unstable, the individual will probably benefit from more
frequent assessment (Grade C; BEL 4; upgraded due to potential benefit).
Jellinger P, Handelsman Y, Rosenblit P, et al. Endocr Practice. 2017;23(4):479-497
R79. Liver transaminase levels should be measured before and 3 months after niacin
or fibric acid treatment initiation because most liver abnormalities occur within 3
months of treatment initiation. Liver transaminase levels should be measured
periodically thereafter (e.g., semiannually or annually) (Grade C; BEL 4; upgraded
due to potential benefit).
R80. Creatine kinase levels should be assessed and the statin discontinued, at least
temporarily, when an individual reports clinically significant myalgias or muscle
weakness on statin therapy (Grade C; BEL 4; upgraded due to potential benefit).
Jellinger P, Handelsman Y, Rosenblit P, et al. Endocr Practice. 2017;23(4):479-
497
R78. More frequent lipid status evaluation is recommended in situations such as
deterioration of diabetes control, use of a new drug known to affect lipid levels,
progression of atherothrombotic disease, considerable weight gain, unexpected
adverse change in any lipid parameter, development of a new ASCVD risk factor, or
convincing new clinical trial evidence or guidelines that suggest stricter lipid goals
(Grade C; BEL 4; upgraded due to potential benefit).
•Lipid Association of India (LAI)
expert consensus statement on
management of dyslipidemia in
Indians 2017: Part 2
SS Iyengar et al. Lipid Association of India (LAI) expert consensus
statement on management of
dyslipidemia in Indians 2017: part 2, Clinical Lipidology, 12:1, 56-109
Focus on Dyslipidaemia and
Comorbidities
Objective of the guideline
 Lipid Association of India (LAI) part 2 recommendations refer to
specific patient populations. These part 2 LAI recommendations
focus on comorbidities associated with Dyslipidemia and also
focuses on specific patient groups.
 These include patients with heart failure, chronic kidney disease,
non-alcoholic fatty liver disease, cerebrovascular disease, thyroid
disorders, inflammatory joint diseases, familial
hypercholesterolemia and human immunodeficiency virus
infection.
 Guideline also consider women, the elderly and post-
transplantation patients.
Dyslipidemia in Heart Failure
 Besides lifestyle measures, statins may be
administered to patients with ischemic HF
with NYHA Class II-III symptoms.
 Patients with advanced symptomatic HF
require individualized care, in these groups,
intensive statin therapy with a goal to achieve
50% reduction in LDL-C levels is justified.
 Statins are also recommended for ischemic HF
patients awaiting heart transplantation.
HF: Heart Failure, LDL-C; Low-density Lipoprotein, NYHA: New York Heart
Association class
Dyslipidemia in Heart Failure
 Statins are not recommended in NYHA Class IV HF.
However, three points can be considered in patients
with advanced HF. Firstly, no excess of side effects.
Secondly, the degree of LDL-C lowering did not
correlate with event rate. Thirdly, there were fewer
hospitalizations
 Statin therapy is not recommended in advanced HF
patients who have a short life expectancy (e.g. because
of comorbidities like malignancy).
 Statins are not recommended in non-ischemic HF or
dilated cardiomyopathy
Dyslipidemia in CKD
 Follow-up measurement of lipid levels should be performed routinely (at treating
physicians’ discretion) in subjects with CKD until the LDL-C and non- HDL-C target
is achieved.
 Therapeutic lifestyle modification should be recommended to all patients.
 A combination of statins/statin plus ezetimibe is recommended for all adults over
the age of 40 years with eGFR < 60 mL/min/1.73 m2.
 Adults between the ages of 18 and 39, with CKD should be treated with statins if
they have one of the following:
– - Known CAD, stroke or peripheral arterial disease
– - Diabetes mellitus
– - Life time risk of ASCVD >30%
Kodachrome Kidney Disease, LDL-C: Low-density Lipoprotein, HDL-C: High-density
Lipoprotein, eGFR: Estimated Glomerular Filtration Rate,, CAD: coronary artery disease,
ASCVD: Atherosclerotic cardiovascular risk factors
Dyslipidemia in CKD
 Patients who are already receiving lipid-lowering therapy at the
time of starting dialysis should continue to receive these agents.
 In adults with dialysis-dependent CKD who are not on statins,
statins could be considered at the lowest dose possible and should
be titrated up carefully to achieve the LDL-C level for very high risk
patients
 All adult kidney transplant recipients should receive statin
therapy.
 Statins should be started at a lower dose in all subjects with eGFR
<30 ml/min with gradual escalation to the recommended dose
along with monitoring for any adverse events
Kodachrome Kidney Disease, LDL-C: Low-density Lipoprotein, HDL-C:
High-density Lipoprotein, eGFR: Estimated Glomerular Filtration Rate,,
CAD: coronary artery disease, ASCVD: Atherosclerotic cardiovascular
Dyslipidemia in CKD
• Renal dosing for commonly available statins
Kodachrome Kidney Disease, LDL-C: Low-density Lipoprotein, HDL-
C: High-density Lipoprotein, eGFR: Estimated Glomerular Filtration
Rate,, CAD: coronary artery disease, ASCVD: Atherosclerotic
cardiovascular risk factors
Dyslipidemia in NAFD
 The available evidence suggests that the risk for
serious liver injury from statins is quite rare and
patients with NAFLD and dyslipidemia are not at an
increased risk for statin-induced hepatotoxicity. Hence,
statins could be used by physicians to treat
dyslipidemia in patients with NAFLD.
 There are few data to suggest usage of statins as a
treatment option for NAFLD. However, it continues to
be a matter of debate and treatment of NAFLD with
statins cannot be recommended at present.
NAFD: Non-alcoholic fatty liver disease
Dyslipidemia in NAFD
 The available evidence suggests that the risk for
serious liver injury from statins is quite rare and
patients with NAFLD and dyslipidemia are not at
an increased risk for statin-induced
hepatotoxicity. Hence, statins could be used by
physicians to treat dyslipidemia in patients with
NAFLD.
 There are few data to suggest usage of statins as
a treatment option for NAFLD. However, it
continues to be a matter of debate and
treatment of NAFLD with statins cannot be
recommended at present.
NAFD: Non-alcoholic fatty liver disease
Dyslipidemia in Cerebrovascular
Disease
 Acute ischemic stroke: Starting a statin is probably
beneficial. Continue statin if patient is already receiving it.
Statins are not contraindicated if patient is thrombolysis. It
may be given by nasogastric tube in patients with dysphagia.
 Acute ICH: Do not initiate statin if patient is not already on
it. Do not discontinue statin if patient is already on a statin.
Optimal blood pressure control is essential in those already
on a statin.
 Primary prevention of stroke: Statin treatment is
recommended in adults with diabetes, or CAD, to lower the
risk of a first stroke.
 Secondary prevention of stroke: Statins are beneficial in
preventing a second ischemic stroke, but do not prevent
fatal stroke.
 Patients with previous haemorrhagic stroke: Statins are
best avoided in patients with previous lobar haemorrhage,
but may be beneficial in preventing stroke in patients with
previous basal-ganglionic haemorrhage who have a high risk
for ischemic events.
ICH: Intracerebral Hemorrahge
 All patients with dyslipidemia should be screened for thyroid dysfunction.
 Patients with dyslipidemia and overt hypothyroidism should be treated
initially with thyroxine.
 In patients with underlying risk for CAD, start low dose thyroxine and
titrate upwards slowly. Assess CVD risk and start lipid-lowering drugs as
per LAI recommendation for primary prevention of CVD.
 Patients without underlying risk for CAD: wait for the patient to become
euthyroid.
 Statins or fibrates alone or in combination in patients with dyslipidemia
and uncontrolled hypothyroidism carry a higher risk of myopathy.
 Thyroid replacement in patients with sub-clinical hypothyroidism if serum
TSH is >10 mIU/L, patients have high initial cholesterol levels or are
elderly, are smokers or are positive for anti-thyroid peroxidase antibodies.
 In patients with established vascular disease requiring thyroxine and lipid-
lowering therapy, caution should be exercised with thyroxine dose to
prevent onset of ischemic symptoms.
Dyslipidemia in Thyroid Disorder
CAD: coronary artery disease, CVD: Cardiovascular disease, TSH: Thyroid stimulating
hormone
 CVD risk assessment for IJD patients should be
carried out [including total cholesterol and
HDL-C (fasting or non-fasting)] during the
stable disease state once every 5 years.
 The use of statins and CVD risk management
should follow the same principles as discussed
in the Part1 of this consensus document.
Management of ASCVD risk in
inflammatory joint diseases (IJD)
CVD: Cardiovascular disease, HDL-C: High density lipoprotein
Familial Hypercholeterolemia (FH)
 Lipid profile estimation of children to be done at 2 years of age
in those with family history of FH and premature ASCVD.
 LAI recommends the Simon Broome criteria for the diagnosis of
FH.
 In an established case of FH, LAI recommends estimation of
Lp(a) levels.
 Genetic testing and cascade screening should be performed
wherever feasible.
 Look for other ASCVD risk factors and manage them
appropriately.
 Strict dietary recommendations and lifestyle modifications as
advised.
 Drug therapy to be started at age 8 years or earlier in
individualised cases.
 LDL-C targets to be achieved:<70 mg/dL for HoFH and <100
mg/dL for HeFH in children and in adults <50 mg/dL in HoFH
and 70 mg/dL in HeFH or at least 50% reduction in LDL-C from
the baseline.
HeFH: Heterozygous FH; HoFH: Homozygous FH; HF: Heart Failure, Lp(a): Lipoprotein,
LDL-C: Low-denisty lipoprotein
Inherited Hypertriglyceridaemia
 All subjects should be screened for
hypertriglyceridaemia with a fasting lipid profile. A
non-fasting lipid profile may be performed as an
initial step, but fasting TG estimation will be needed
if TG is found to be high in the non-fasting sample
 Exclude and treat secondary causes
 Aggressive therapeutic lifestyle changes should be
implemented
 Subjects with TG > 200 mg/dL and <500 mg/dL –
treat with a statin
 Subjects with TG ≥ 500 mg/dL-treat with fibrates to
prevent acute pancreatitis and later add statin to
achieve non-HDL-C goal
 Preferred drugs: Fibrates and high dose omega fatty
acids; the role of niacin is controversial.
TG; Triglycerides, HDL-C: High-density lipoprotein
Lipids targets in HIV patients as recommended by
LAI
Dyslipidemia in HIV
Effect of different statins in HIV patients with
dyslipidemia
LDL-C: Low-denisty lipoprotein, HDL-C: High density
 Lifestyle modifications with special emphasis on
smoking cessation, weight reduction and calorie
restriction are important.
 The recommendations for the evaluation and
treatment of dyslipidemia as suggested by LAI
regarding target goals for lipids should be strictly
followed to help guide therapy.
 At least, evidence available in the Indian
population suggests the same.
Dyslipidemia in HIV
HIV: Human Immunodeficiency Virus, LDL-C: Low-density lipoprotein
 Different types of statins are available to lower
plasma lipids to guideline levels in patients with HIV,
but they differ in their pharmacokinetic properties
and drug interaction profiles. Simvastatin and
lovastatin are contraindicated in patients taking PIs.
 The other statins atorvastatin and rosuvastatin
(apart from simvastatin and lovastatin) have better
therapeutic effect in lowering LDL-C in HIV
dyslipidemia. The addition of ezetimibe is another
option. Fenofibrate and fish oil can be used in statin-
unresponsive HIV dyslipidemia.
Dyslipidemia in HIV
HIV: Human Immunodeficiency Virus, LDL-C: Low-density lipoprotein
Dyslipidemia in Women
 Women with collagen vascular disorders, PCOS,
preeclampsia, pregnancy-induced hypertension and
gestational diabetes are at higher ASCVD risk.
 The dyslipidemia management is similar in both men and
women. However, in elderly frail women, treatment should
be started with low dose statin in view of higher incidence of
muscle-related adverse effects except in the very high risk
group.
 Statins remain the first-line cholesterol-lowering drug
therapy for primary and secondary prevention in women.
Statin dosage should be increased to the maximally tolerated
dose before adding a non-statin drug if goal levels of LDL-C
and non-HDL-C are not achieved.
 Non-statin drugs may be considered as a primary drug for
women with contraindications for, or intolerance to, statin
therapy.
PCOS: Polycystic ovarian syndrome, LDL-C: Low-denisty lipoprotein, HDL-C:
High density lipoprotein
Dyslipidemia in Elderly
 CVD is the most common cause of death in the elderly. As in
the young, elderly patients with dyslipidemia have an
increased risk for CVD. • Secondary causes of dyslipidemia
such as hypothyroidism, diabetes, CKD and drug effects
should be considered in elderly patients.
 The RRR of lipid-lowering therapy in elderly patients is similar
to that in younger patients; however, the absolute benefit is
higher than in younger patients.
 Reductions in events with statin therapy are apparent within
a few weeks to months even in older patients.
RRR: Relative Risk Reduction, CKD: Chronic kidney disease, CVD: Cardiovascular
Dyslipidemia in Elderly
 Side effects of lipid-lowering therapy are similar in the old
and young. However, in very elderly frail patients, lower
doses may be appropriate.
 Secondary prevention: Statin therapy should be instituted in
all appropriate elderly patients as above with treatment goals
similar to younger patients. Dose of statins may need to be
individualized in the elderly based on frailty and other
comorbidities.
 Primary prevention: Elderly subjects with a reasonable life
expectancy should be treated similar to younger patients as
per risk stratification algorithm proposed in the previously
published LAI expert consensus statement
RRR: Relative Risk Reduction, CKD: Chronic kidney disease, CVD: Cardiovascular
Post-Heart Transplantation
 Baseline lipid levels should be obtained for all
patients after heart transplantation.
 The strict control of modifiable risk factors
including hypertension, diabetes, dyslipidemia,
smoking and obesity should be reinforced.
 In adults, the use of statins beginning 1 to 2
weeks after heart transplantation is
recommended regardless of cholesterol levels.
CAV: Cardiac Allograft Vasculopathy, LDL-C: Low-denisty
Post-Heart Transplantation
 Pravastatin shows the least interaction with
cyclosporine to produce myopathy, making it the
drug of choice.
 Rosuvastatin up to 10 mg/day may be given as a
second choice.
 Addition of a statin to a cyclosporine–sirolimus
regimen produces multiple beneficial effects like
reduced cholesterol levels, decreased acute
rejection episodes, decreased incidence of CAV
and improved survival.
 In heart transplant recipients, a strategy of
lowest achievable LDL-C levels with maximally
tolerated dose of statin by slowly up-titrating the
statin dose is justified.
CAV: Cardiac Allograft Vasculopathy, LDL-C: Low-denisty
Sources
• Stone Nj, Robinson J, Lichtenstein Ah, Bairey Merz Cn,
Lioyd-jones Dm, Blum Cb, Mcbride P, eckel Rh, Schwartz Js,
Goldberg Ac, Shero St, Gordon D, Smith Sc Jr, Levy D,
Watson K, Wilson Pw. 2013 ACC/AHA Guideline On The
Treatment Of Blood Cholesterol To Reduce Atherosclerotic
Cardiovascular Risk In Adults: A Report Of The American
College Of Cardiology/American Heart Association Task
Force On Practice Guidelines. J Am Coll Cardiol. 2013 Nov 7.
Pii: S0735-1097
• John F. Keaney, Jr., M.D., Gregory D. Curfman, M.D., And
John A. Jarcho, M.D. A Pragmatic View Of The New
Cholesterol Treatment Guidelines. N Engl J Med 2014;
370:275-278
Thank you !

Weitere ähnliche Inhalte

Was ist angesagt?

Diabetic dyslipidemia
Diabetic dyslipidemiaDiabetic dyslipidemia
Diabetic dyslipidemiaFarragBahbah
 
Dyslipidemia by dr. topu
Dyslipidemia by dr. topuDyslipidemia by dr. topu
Dyslipidemia by dr. topuNizam Uddin
 
DIABETES AND CARDIOVASCULAR DISEASE - THE CONTINUUM
DIABETES AND CARDIOVASCULAR DISEASE - THE CONTINUUMDIABETES AND CARDIOVASCULAR DISEASE - THE CONTINUUM
DIABETES AND CARDIOVASCULAR DISEASE - THE CONTINUUMPraveen Nagula
 
DYSLIPIDEMIA GUIDELINES
DYSLIPIDEMIA GUIDELINESDYSLIPIDEMIA GUIDELINES
DYSLIPIDEMIA GUIDELINESarnab ghosh
 
LDL Cholesterol Target :“ Lower the Better ”
LDL Cholesterol Target :“ Lower the Better ”LDL Cholesterol Target :“ Lower the Better ”
LDL Cholesterol Target :“ Lower the Better ”Arindam Pande
 
Acc 2018 guidelines on lipids
Acc 2018 guidelines on lipidsAcc 2018 guidelines on lipids
Acc 2018 guidelines on lipidsDr Anu Grover
 
Aace Guideline 2017: Management of Dyslipidemia and Prevention of Atheroscle...
Aace Guideline 2017:  Management of Dyslipidemia and Prevention of Atheroscle...Aace Guideline 2017:  Management of Dyslipidemia and Prevention of Atheroscle...
Aace Guideline 2017: Management of Dyslipidemia and Prevention of Atheroscle...Syed Mogni
 
ACC/AHA lipid guidelines 2018
ACC/AHA lipid guidelines 2018ACC/AHA lipid guidelines 2018
ACC/AHA lipid guidelines 2018Mgfamiliar Net
 
Metabolic Syndrome, Diabetes, and Cardiovascular Disease ... Metabolic Synd...
Metabolic Syndrome, Diabetes, and Cardiovascular Disease ... 	 Metabolic Synd...Metabolic Syndrome, Diabetes, and Cardiovascular Disease ... 	 Metabolic Synd...
Metabolic Syndrome, Diabetes, and Cardiovascular Disease ... Metabolic Synd...MedicineAndFamily
 
Current Controversies in Dyslipidemia Management:
Current Controversies in Dyslipidemia Management:Current Controversies in Dyslipidemia Management:
Current Controversies in Dyslipidemia Management:magdy elmasry
 
Novel approaches in Lipid Management
Novel approaches in Lipid ManagementNovel approaches in Lipid Management
Novel approaches in Lipid ManagementShashikiran Umakanth
 
Diabetic dyslipidemia and Saroglitazar
Diabetic dyslipidemia and SaroglitazarDiabetic dyslipidemia and Saroglitazar
Diabetic dyslipidemia and SaroglitazarDr Vivek Baliga
 

Was ist angesagt? (20)

2019 ESC/EAS Guidelines on Dyslipidaemias
2019 ESC/EAS Guidelines on Dyslipidaemias2019 ESC/EAS Guidelines on Dyslipidaemias
2019 ESC/EAS Guidelines on Dyslipidaemias
 
The ESC/EAS Guidelines
The ESC/EAS GuidelinesThe ESC/EAS Guidelines
The ESC/EAS Guidelines
 
Diabetic dyslipidemia
Diabetic dyslipidemiaDiabetic dyslipidemia
Diabetic dyslipidemia
 
dyslipidemia6.ppt
dyslipidemia6.pptdyslipidemia6.ppt
dyslipidemia6.ppt
 
Dyslipidemia by dr. topu
Dyslipidemia by dr. topuDyslipidemia by dr. topu
Dyslipidemia by dr. topu
 
DIABETES AND CARDIOVASCULAR DISEASE - THE CONTINUUM
DIABETES AND CARDIOVASCULAR DISEASE - THE CONTINUUMDIABETES AND CARDIOVASCULAR DISEASE - THE CONTINUUM
DIABETES AND CARDIOVASCULAR DISEASE - THE CONTINUUM
 
DYSLIPIDEMIA GUIDELINES
DYSLIPIDEMIA GUIDELINESDYSLIPIDEMIA GUIDELINES
DYSLIPIDEMIA GUIDELINES
 
LDL Cholesterol Target :“ Lower the Better ”
LDL Cholesterol Target :“ Lower the Better ”LDL Cholesterol Target :“ Lower the Better ”
LDL Cholesterol Target :“ Lower the Better ”
 
Acc 2018 guidelines on lipids
Acc 2018 guidelines on lipidsAcc 2018 guidelines on lipids
Acc 2018 guidelines on lipids
 
Diabetic Dyslipidemia- Dr Shahjada Selim
Diabetic Dyslipidemia- Dr Shahjada SelimDiabetic Dyslipidemia- Dr Shahjada Selim
Diabetic Dyslipidemia- Dr Shahjada Selim
 
Aace Guideline 2017: Management of Dyslipidemia and Prevention of Atheroscle...
Aace Guideline 2017:  Management of Dyslipidemia and Prevention of Atheroscle...Aace Guideline 2017:  Management of Dyslipidemia and Prevention of Atheroscle...
Aace Guideline 2017: Management of Dyslipidemia and Prevention of Atheroscle...
 
ACC/AHA lipid guidelines 2018
ACC/AHA lipid guidelines 2018ACC/AHA lipid guidelines 2018
ACC/AHA lipid guidelines 2018
 
Metabolic Syndrome, Diabetes, and Cardiovascular Disease ... Metabolic Synd...
Metabolic Syndrome, Diabetes, and Cardiovascular Disease ... 	 Metabolic Synd...Metabolic Syndrome, Diabetes, and Cardiovascular Disease ... 	 Metabolic Synd...
Metabolic Syndrome, Diabetes, and Cardiovascular Disease ... Metabolic Synd...
 
Role of SGLT2i in cardio-renal protection
Role of SGLT2i in cardio-renal protectionRole of SGLT2i in cardio-renal protection
Role of SGLT2i in cardio-renal protection
 
Pcsk 9 inhibitors
Pcsk 9 inhibitorsPcsk 9 inhibitors
Pcsk 9 inhibitors
 
Current Controversies in Dyslipidemia Management:
Current Controversies in Dyslipidemia Management:Current Controversies in Dyslipidemia Management:
Current Controversies in Dyslipidemia Management:
 
Novel approaches in Lipid Management
Novel approaches in Lipid ManagementNovel approaches in Lipid Management
Novel approaches in Lipid Management
 
Dyslipidemia
DyslipidemiaDyslipidemia
Dyslipidemia
 
Ontarget
OntargetOntarget
Ontarget
 
Diabetic dyslipidemia and Saroglitazar
Diabetic dyslipidemia and SaroglitazarDiabetic dyslipidemia and Saroglitazar
Diabetic dyslipidemia and Saroglitazar
 

Ähnlich wie lipid guidelines.pptx

2. prof. bambang irawan cv assessment in met s and t2dm [compatibility mode]
2. prof. bambang irawan cv assessment in met s and t2dm [compatibility mode]2. prof. bambang irawan cv assessment in met s and t2dm [compatibility mode]
2. prof. bambang irawan cv assessment in met s and t2dm [compatibility mode]yoga buana
 
Cap nhat lipid 2017
Cap nhat lipid 2017Cap nhat lipid 2017
Cap nhat lipid 2017khacleson
 
Dyslipidemia and drug resistant dyslipidemia
Dyslipidemia and drug resistant dyslipidemiaDyslipidemia and drug resistant dyslipidemia
Dyslipidemia and drug resistant dyslipidemiaDr Siva subramaniyan
 
Ueda2015 d.dyslipidemia dr.khaled hadidy
Ueda2015 d.dyslipidemia dr.khaled hadidyUeda2015 d.dyslipidemia dr.khaled hadidy
Ueda2015 d.dyslipidemia dr.khaled hadidyueda2015
 
Dyslipidaemia presentation
Dyslipidaemia presentationDyslipidaemia presentation
Dyslipidaemia presentationrajeetam123
 
Targeting lipids: a primary and secondary care perspective
Targeting lipids: a primary and secondary care perspectiveTargeting lipids: a primary and secondary care perspective
Targeting lipids: a primary and secondary care perspectiveInnovation Agency
 
Dyslipidaemia part one.pdf
Dyslipidaemia part one.pdfDyslipidaemia part one.pdf
Dyslipidaemia part one.pdfssuser0ec0d3
 
Old vs new targets april 2015
Old vs new targets april 2015Old vs new targets april 2015
Old vs new targets april 2015Henry Tran
 
Dyslipidemia GL & Total Vascular Benefit .pptx
Dyslipidemia GL & Total Vascular Benefit .pptxDyslipidemia GL & Total Vascular Benefit .pptx
Dyslipidemia GL & Total Vascular Benefit .pptxWidiHadian3
 

Ähnlich wie lipid guidelines.pptx (20)

Lipid guidelines
Lipid guidelinesLipid guidelines
Lipid guidelines
 
Dyslipidaemia highlights
Dyslipidaemia highlights Dyslipidaemia highlights
Dyslipidaemia highlights
 
Dyslipidemia Guidlines
Dyslipidemia GuidlinesDyslipidemia Guidlines
Dyslipidemia Guidlines
 
2. prof. bambang irawan cv assessment in met s and t2dm [compatibility mode]
2. prof. bambang irawan cv assessment in met s and t2dm [compatibility mode]2. prof. bambang irawan cv assessment in met s and t2dm [compatibility mode]
2. prof. bambang irawan cv assessment in met s and t2dm [compatibility mode]
 
Cap nhat lipid 2017
Cap nhat lipid 2017Cap nhat lipid 2017
Cap nhat lipid 2017
 
Dyslipidemia
DyslipidemiaDyslipidemia
Dyslipidemia
 
Dyslipidemia and drug resistant dyslipidemia
Dyslipidemia and drug resistant dyslipidemiaDyslipidemia and drug resistant dyslipidemia
Dyslipidemia and drug resistant dyslipidemia
 
Dyslipidemia overview 2017
Dyslipidemia overview 2017Dyslipidemia overview 2017
Dyslipidemia overview 2017
 
What after metformin ?
What after metformin ? What after metformin ?
What after metformin ?
 
Ueda2015 d.dyslipidemia dr.khaled hadidy
Ueda2015 d.dyslipidemia dr.khaled hadidyUeda2015 d.dyslipidemia dr.khaled hadidy
Ueda2015 d.dyslipidemia dr.khaled hadidy
 
statin.pptx
statin.pptxstatin.pptx
statin.pptx
 
Dyslipidaemia presentation
Dyslipidaemia presentationDyslipidaemia presentation
Dyslipidaemia presentation
 
Targeting lipids: a primary and secondary care perspective
Targeting lipids: a primary and secondary care perspectiveTargeting lipids: a primary and secondary care perspective
Targeting lipids: a primary and secondary care perspective
 
NPCDCS BY DR.R.MOHAN
NPCDCS  BY  DR.R.MOHANNPCDCS  BY  DR.R.MOHAN
NPCDCS BY DR.R.MOHAN
 
Dyslipidaemia part one.pdf
Dyslipidaemia part one.pdfDyslipidaemia part one.pdf
Dyslipidaemia part one.pdf
 
Hypertension
HypertensionHypertension
Hypertension
 
Dyslipidemia aha acc 2013
Dyslipidemia aha acc 2013Dyslipidemia aha acc 2013
Dyslipidemia aha acc 2013
 
Old vs new targets april 2015
Old vs new targets april 2015Old vs new targets april 2015
Old vs new targets april 2015
 
Dyslipidemia GL & Total Vascular Benefit .pptx
Dyslipidemia GL & Total Vascular Benefit .pptxDyslipidemia GL & Total Vascular Benefit .pptx
Dyslipidemia GL & Total Vascular Benefit .pptx
 
Diabetic Dyslipidemia Slide Share
Diabetic  Dyslipidemia Slide ShareDiabetic  Dyslipidemia Slide Share
Diabetic Dyslipidemia Slide Share
 

Mehr von Sruthi Meenaxshi

MEN SYNDROMESAND GENETIC BASIS.pptx
MEN SYNDROMESAND GENETIC BASIS.pptxMEN SYNDROMESAND GENETIC BASIS.pptx
MEN SYNDROMESAND GENETIC BASIS.pptxSruthi Meenaxshi
 
ADULT NEURODEGENERATIVE DISORDERS .pptx
ADULT NEURODEGENERATIVE DISORDERS .pptxADULT NEURODEGENERATIVE DISORDERS .pptx
ADULT NEURODEGENERATIVE DISORDERS .pptxSruthi Meenaxshi
 
DRUG INDUCED LIVER INJURY.pptx
DRUG INDUCED LIVER INJURY.pptxDRUG INDUCED LIVER INJURY.pptx
DRUG INDUCED LIVER INJURY.pptxSruthi Meenaxshi
 
acute hiv inffection and cdc criteria.pptx
acute hiv inffection and cdc criteria.pptxacute hiv inffection and cdc criteria.pptx
acute hiv inffection and cdc criteria.pptxSruthi Meenaxshi
 
head ache dizziness and sphincter disturbance s.pptx
head ache dizziness and sphincter disturbance s.pptxhead ache dizziness and sphincter disturbance s.pptx
head ache dizziness and sphincter disturbance s.pptxSruthi Meenaxshi
 
inflammatory bowel disease.pptx
inflammatory bowel disease.pptxinflammatory bowel disease.pptx
inflammatory bowel disease.pptxSruthi Meenaxshi
 
Arrythmogenic rv dysplasia (ARVD)
Arrythmogenic rv dysplasia (ARVD)Arrythmogenic rv dysplasia (ARVD)
Arrythmogenic rv dysplasia (ARVD)Sruthi Meenaxshi
 
Left ventricular noncompaction
Left ventricular noncompactionLeft ventricular noncompaction
Left ventricular noncompactionSruthi Meenaxshi
 
Vector borne diseases recent concepts in management and elimination targets...
Vector borne diseases   recent concepts in management and elimination targets...Vector borne diseases   recent concepts in management and elimination targets...
Vector borne diseases recent concepts in management and elimination targets...Sruthi Meenaxshi
 
Echo in prosthetic valve evaluation
Echo in prosthetic valve evaluationEcho in prosthetic valve evaluation
Echo in prosthetic valve evaluationSruthi Meenaxshi
 
Ventricular Septal defects Echocardiography
Ventricular Septal defects EchocardiographyVentricular Septal defects Echocardiography
Ventricular Septal defects EchocardiographySruthi Meenaxshi
 
Atrial septal defect Echocardiography
Atrial septal defect EchocardiographyAtrial septal defect Echocardiography
Atrial septal defect EchocardiographySruthi Meenaxshi
 
Infective endocarditis Echocardiography
Infective endocarditis EchocardiographyInfective endocarditis Echocardiography
Infective endocarditis EchocardiographySruthi Meenaxshi
 
Conduction abnormalities part 2
Conduction abnormalities part 2Conduction abnormalities part 2
Conduction abnormalities part 2Sruthi Meenaxshi
 

Mehr von Sruthi Meenaxshi (20)

MEN SYNDROMESAND GENETIC BASIS.pptx
MEN SYNDROMESAND GENETIC BASIS.pptxMEN SYNDROMESAND GENETIC BASIS.pptx
MEN SYNDROMESAND GENETIC BASIS.pptx
 
ADULT NEURODEGENERATIVE DISORDERS .pptx
ADULT NEURODEGENERATIVE DISORDERS .pptxADULT NEURODEGENERATIVE DISORDERS .pptx
ADULT NEURODEGENERATIVE DISORDERS .pptx
 
DRUG INDUCED LIVER INJURY.pptx
DRUG INDUCED LIVER INJURY.pptxDRUG INDUCED LIVER INJURY.pptx
DRUG INDUCED LIVER INJURY.pptx
 
AORTIC STENOSIS.pptx
AORTIC STENOSIS.pptxAORTIC STENOSIS.pptx
AORTIC STENOSIS.pptx
 
drugoverdose.pptx
drugoverdose.pptxdrugoverdose.pptx
drugoverdose.pptx
 
scorpion envenomation.pptx
scorpion envenomation.pptxscorpion envenomation.pptx
scorpion envenomation.pptx
 
acute hiv inffection and cdc criteria.pptx
acute hiv inffection and cdc criteria.pptxacute hiv inffection and cdc criteria.pptx
acute hiv inffection and cdc criteria.pptx
 
head ache dizziness and sphincter disturbance s.pptx
head ache dizziness and sphincter disturbance s.pptxhead ache dizziness and sphincter disturbance s.pptx
head ache dizziness and sphincter disturbance s.pptx
 
inflammatory bowel disease.pptx
inflammatory bowel disease.pptxinflammatory bowel disease.pptx
inflammatory bowel disease.pptx
 
Ventricular arrythmia
Ventricular arrythmiaVentricular arrythmia
Ventricular arrythmia
 
Digoxin toxicity
Digoxin toxicityDigoxin toxicity
Digoxin toxicity
 
Arrythmogenic rv dysplasia (ARVD)
Arrythmogenic rv dysplasia (ARVD)Arrythmogenic rv dysplasia (ARVD)
Arrythmogenic rv dysplasia (ARVD)
 
Left ventricular noncompaction
Left ventricular noncompactionLeft ventricular noncompaction
Left ventricular noncompaction
 
Vector borne diseases recent concepts in management and elimination targets...
Vector borne diseases   recent concepts in management and elimination targets...Vector borne diseases   recent concepts in management and elimination targets...
Vector borne diseases recent concepts in management and elimination targets...
 
Asd device closure
Asd device closureAsd device closure
Asd device closure
 
Echo in prosthetic valve evaluation
Echo in prosthetic valve evaluationEcho in prosthetic valve evaluation
Echo in prosthetic valve evaluation
 
Ventricular Septal defects Echocardiography
Ventricular Septal defects EchocardiographyVentricular Septal defects Echocardiography
Ventricular Septal defects Echocardiography
 
Atrial septal defect Echocardiography
Atrial septal defect EchocardiographyAtrial septal defect Echocardiography
Atrial septal defect Echocardiography
 
Infective endocarditis Echocardiography
Infective endocarditis EchocardiographyInfective endocarditis Echocardiography
Infective endocarditis Echocardiography
 
Conduction abnormalities part 2
Conduction abnormalities part 2Conduction abnormalities part 2
Conduction abnormalities part 2
 

Kürzlich hochgeladen

BENIGN BREAST DISEASE
BENIGN BREAST DISEASE BENIGN BREAST DISEASE
BENIGN BREAST DISEASE Mamatha Lakka
 
CONNECTIVE TISSUE (ANATOMY AND PHYSIOLOGY).pdf
CONNECTIVE TISSUE (ANATOMY AND PHYSIOLOGY).pdfCONNECTIVE TISSUE (ANATOMY AND PHYSIOLOGY).pdf
CONNECTIVE TISSUE (ANATOMY AND PHYSIOLOGY).pdfDolisha Warbi
 
SGK ĐIỆN GIẬT ĐHYHN RẤT LÀ HAY TUYỆT VỜI.pdf
SGK ĐIỆN GIẬT ĐHYHN        RẤT LÀ HAY TUYỆT VỜI.pdfSGK ĐIỆN GIẬT ĐHYHN        RẤT LÀ HAY TUYỆT VỜI.pdf
SGK ĐIỆN GIẬT ĐHYHN RẤT LÀ HAY TUYỆT VỜI.pdfHongBiThi1
 
Breast cancer -ONCO IN MEDICAL AND SURGICAL NURSING.pptx
Breast cancer -ONCO IN MEDICAL AND SURGICAL NURSING.pptxBreast cancer -ONCO IN MEDICAL AND SURGICAL NURSING.pptx
Breast cancer -ONCO IN MEDICAL AND SURGICAL NURSING.pptxNaveenkumar267201
 
Role of Soap based and synthetic or syndets bar
Role of  Soap based and synthetic or syndets barRole of  Soap based and synthetic or syndets bar
Role of Soap based and synthetic or syndets barmohitRahangdale
 
Physiology of Smooth Muscles -Mechanics of contraction and relaxation
Physiology of Smooth Muscles -Mechanics of contraction and relaxationPhysiology of Smooth Muscles -Mechanics of contraction and relaxation
Physiology of Smooth Muscles -Mechanics of contraction and relaxationMedicoseAcademics
 
power point presentation of Clinical evaluation of strabismus
power point presentation of Clinical evaluation  of strabismuspower point presentation of Clinical evaluation  of strabismus
power point presentation of Clinical evaluation of strabismusChandrasekar Reddy
 
Trustworthiness of AI based predictions Aachen 2024
Trustworthiness of AI based predictions Aachen 2024Trustworthiness of AI based predictions Aachen 2024
Trustworthiness of AI based predictions Aachen 2024EwoutSteyerberg1
 
SGK RỐI LOẠN TOAN KIỀM ĐHYHN RẤT HAY VÀ ĐẶC SẮC.pdf
SGK RỐI LOẠN TOAN KIỀM ĐHYHN RẤT HAY VÀ ĐẶC SẮC.pdfSGK RỐI LOẠN TOAN KIỀM ĐHYHN RẤT HAY VÀ ĐẶC SẮC.pdf
SGK RỐI LOẠN TOAN KIỀM ĐHYHN RẤT HAY VÀ ĐẶC SẮC.pdfHongBiThi1
 
How to cure cirrhosis and chronic hepatitis naturally
How to cure cirrhosis and chronic hepatitis naturallyHow to cure cirrhosis and chronic hepatitis naturally
How to cure cirrhosis and chronic hepatitis naturallyZurück zum Ursprung
 
Generative AI in Health Care a scoping review and a persoanl experience.
Generative AI in Health Care a scoping review and a persoanl experience.Generative AI in Health Care a scoping review and a persoanl experience.
Generative AI in Health Care a scoping review and a persoanl experience.Vaikunthan Rajaratnam
 
FDMA FLAP - The first dorsal metacarpal artery (FDMA) flap is used mainly for...
FDMA FLAP - The first dorsal metacarpal artery (FDMA) flap is used mainly for...FDMA FLAP - The first dorsal metacarpal artery (FDMA) flap is used mainly for...
FDMA FLAP - The first dorsal metacarpal artery (FDMA) flap is used mainly for...Shubhanshu Gaurav
 
CPR.nursingoutlook.pdf , Bsc nursing student
CPR.nursingoutlook.pdf , Bsc nursing studentCPR.nursingoutlook.pdf , Bsc nursing student
CPR.nursingoutlook.pdf , Bsc nursing studentsaileshpanda05
 
MedMatch: Your Health, Our Mission. Pitch deck.
MedMatch: Your Health, Our Mission. Pitch deck.MedMatch: Your Health, Our Mission. Pitch deck.
MedMatch: Your Health, Our Mission. Pitch deck.whalesdesign
 
Male Infertility Panel Discussion by Dr Sujoy Dasgupta
Male Infertility Panel Discussion by Dr Sujoy DasguptaMale Infertility Panel Discussion by Dr Sujoy Dasgupta
Male Infertility Panel Discussion by Dr Sujoy DasguptaSujoy Dasgupta
 
Using Data Visualization in Public Health Communications
Using Data Visualization in Public Health CommunicationsUsing Data Visualization in Public Health Communications
Using Data Visualization in Public Health Communicationskatiequigley33
 
Bulimia nervosa ( Eating Disorders) Mental Health Nursing.
Bulimia nervosa ( Eating Disorders) Mental Health Nursing.Bulimia nervosa ( Eating Disorders) Mental Health Nursing.
Bulimia nervosa ( Eating Disorders) Mental Health Nursing.aarjukhadka22
 
Clinical Research Informatics Year-in-Review 2024
Clinical Research Informatics Year-in-Review 2024Clinical Research Informatics Year-in-Review 2024
Clinical Research Informatics Year-in-Review 2024Peter Embi
 
Different drug regularity bodies in different countries.
Different drug regularity bodies in different countries.Different drug regularity bodies in different countries.
Different drug regularity bodies in different countries.kishan singh tomar
 
Female Reproductive Physiology Before Pregnancy
Female Reproductive Physiology Before PregnancyFemale Reproductive Physiology Before Pregnancy
Female Reproductive Physiology Before PregnancyMedicoseAcademics
 

Kürzlich hochgeladen (20)

BENIGN BREAST DISEASE
BENIGN BREAST DISEASE BENIGN BREAST DISEASE
BENIGN BREAST DISEASE
 
CONNECTIVE TISSUE (ANATOMY AND PHYSIOLOGY).pdf
CONNECTIVE TISSUE (ANATOMY AND PHYSIOLOGY).pdfCONNECTIVE TISSUE (ANATOMY AND PHYSIOLOGY).pdf
CONNECTIVE TISSUE (ANATOMY AND PHYSIOLOGY).pdf
 
SGK ĐIỆN GIẬT ĐHYHN RẤT LÀ HAY TUYỆT VỜI.pdf
SGK ĐIỆN GIẬT ĐHYHN        RẤT LÀ HAY TUYỆT VỜI.pdfSGK ĐIỆN GIẬT ĐHYHN        RẤT LÀ HAY TUYỆT VỜI.pdf
SGK ĐIỆN GIẬT ĐHYHN RẤT LÀ HAY TUYỆT VỜI.pdf
 
Breast cancer -ONCO IN MEDICAL AND SURGICAL NURSING.pptx
Breast cancer -ONCO IN MEDICAL AND SURGICAL NURSING.pptxBreast cancer -ONCO IN MEDICAL AND SURGICAL NURSING.pptx
Breast cancer -ONCO IN MEDICAL AND SURGICAL NURSING.pptx
 
Role of Soap based and synthetic or syndets bar
Role of  Soap based and synthetic or syndets barRole of  Soap based and synthetic or syndets bar
Role of Soap based and synthetic or syndets bar
 
Physiology of Smooth Muscles -Mechanics of contraction and relaxation
Physiology of Smooth Muscles -Mechanics of contraction and relaxationPhysiology of Smooth Muscles -Mechanics of contraction and relaxation
Physiology of Smooth Muscles -Mechanics of contraction and relaxation
 
power point presentation of Clinical evaluation of strabismus
power point presentation of Clinical evaluation  of strabismuspower point presentation of Clinical evaluation  of strabismus
power point presentation of Clinical evaluation of strabismus
 
Trustworthiness of AI based predictions Aachen 2024
Trustworthiness of AI based predictions Aachen 2024Trustworthiness of AI based predictions Aachen 2024
Trustworthiness of AI based predictions Aachen 2024
 
SGK RỐI LOẠN TOAN KIỀM ĐHYHN RẤT HAY VÀ ĐẶC SẮC.pdf
SGK RỐI LOẠN TOAN KIỀM ĐHYHN RẤT HAY VÀ ĐẶC SẮC.pdfSGK RỐI LOẠN TOAN KIỀM ĐHYHN RẤT HAY VÀ ĐẶC SẮC.pdf
SGK RỐI LOẠN TOAN KIỀM ĐHYHN RẤT HAY VÀ ĐẶC SẮC.pdf
 
How to cure cirrhosis and chronic hepatitis naturally
How to cure cirrhosis and chronic hepatitis naturallyHow to cure cirrhosis and chronic hepatitis naturally
How to cure cirrhosis and chronic hepatitis naturally
 
Generative AI in Health Care a scoping review and a persoanl experience.
Generative AI in Health Care a scoping review and a persoanl experience.Generative AI in Health Care a scoping review and a persoanl experience.
Generative AI in Health Care a scoping review and a persoanl experience.
 
FDMA FLAP - The first dorsal metacarpal artery (FDMA) flap is used mainly for...
FDMA FLAP - The first dorsal metacarpal artery (FDMA) flap is used mainly for...FDMA FLAP - The first dorsal metacarpal artery (FDMA) flap is used mainly for...
FDMA FLAP - The first dorsal metacarpal artery (FDMA) flap is used mainly for...
 
CPR.nursingoutlook.pdf , Bsc nursing student
CPR.nursingoutlook.pdf , Bsc nursing studentCPR.nursingoutlook.pdf , Bsc nursing student
CPR.nursingoutlook.pdf , Bsc nursing student
 
MedMatch: Your Health, Our Mission. Pitch deck.
MedMatch: Your Health, Our Mission. Pitch deck.MedMatch: Your Health, Our Mission. Pitch deck.
MedMatch: Your Health, Our Mission. Pitch deck.
 
Male Infertility Panel Discussion by Dr Sujoy Dasgupta
Male Infertility Panel Discussion by Dr Sujoy DasguptaMale Infertility Panel Discussion by Dr Sujoy Dasgupta
Male Infertility Panel Discussion by Dr Sujoy Dasgupta
 
Using Data Visualization in Public Health Communications
Using Data Visualization in Public Health CommunicationsUsing Data Visualization in Public Health Communications
Using Data Visualization in Public Health Communications
 
Bulimia nervosa ( Eating Disorders) Mental Health Nursing.
Bulimia nervosa ( Eating Disorders) Mental Health Nursing.Bulimia nervosa ( Eating Disorders) Mental Health Nursing.
Bulimia nervosa ( Eating Disorders) Mental Health Nursing.
 
Clinical Research Informatics Year-in-Review 2024
Clinical Research Informatics Year-in-Review 2024Clinical Research Informatics Year-in-Review 2024
Clinical Research Informatics Year-in-Review 2024
 
Different drug regularity bodies in different countries.
Different drug regularity bodies in different countries.Different drug regularity bodies in different countries.
Different drug regularity bodies in different countries.
 
Female Reproductive Physiology Before Pregnancy
Female Reproductive Physiology Before PregnancyFemale Reproductive Physiology Before Pregnancy
Female Reproductive Physiology Before Pregnancy
 

lipid guidelines.pptx

  • 2. Dyslipidemia – Definition • is, quite simply “abnormal lipid levels”, as measured on a blood sample and which reflects one of several disorders in the metabolism of lipoproteins. It may be classified as: • hypercholesterolemia • low levels of High Density Lipoproteins (HDL) • hypertriglyceridemia
  • 3. ATP III Classification LDL cholesterol <100 Optimal 100-129 Near optimal/above optimal 130-159 Borderline high 160-189 high >190 Very high Total cholestrol <200 200-239 >240 HDL cholestrol >40 >60 Desirable Borderline high High Low high
  • 12. Question 1 41 yo M evaluated in PCP appointment. He is healthy with no symptoms, is sedentary and obese. PMH and FHx non-contributory. He does not smoke, drink, or use illicit drugs. He takes no meds. On exam, BP 132/82, HR 80, and BMI 32. Labs show total cholesterol 251, LDL 172, HDL 35, TG 220, HbA1c 5%. Estimated 10 year risk for ASCVD is 3.4%
  • 13. Which of the following is the most appropriate management of this patient’s hyperlipidemia? A: Ezetimibe B: High-intensity rosuvastatin C: Moderate intensity rosuvastatin D. Niacin E: Therapeutic Lifestyle modification
  • 14. Question 2 80 yo M was hospitalized for a 5 day history of acute leg ischemia treated with angioplasty and stenting. Now asymptomatic. PMH significant for CKD stage III and HTN on diltiazem, lisinopril, ASA and plavix. Exam is normal. Labs: AST 20, Total cholesterol 170, LDL 97, HDL 44, Cr. 1.8, TG 147 and GFR 35
  • 15. Question 2 Which of the following is the most appropriate therapy for secondary prevention of cardiovascular disease in this patient? A: High intensity rosuvastatin B: Moderate intensity rousvastatin C: Niacin D: No additional Treatment
  • 16. Question 3 48 yo M evaluated during follow up appointment. 3 months ago, he had a STEMI and underwent PCI with bare metal stent of Left circumflex artery. He was started on high intensity rosuvastatin at the time. LFT’s normal and Cr level was normal. He is now asymptomatic, no chest pain or muscle pain.His meds are aspirin, metoprolol, lisinopril, rosuvastatin, and plavix. Exam and vitals normal. No muscle or abdominal tenderness.
  • 17. Question 3 Which of the following is the most appropriate laboratory study to obtain at this visit? A: Alanine aminotransferase level B: Creatine Kinase Level C: Fasting Lipid Panel D. High sensitivity C-reactive protein level
  • 18. Introduction • Treatment of dyslipidemia is a cornerstone of preventive cardiology, and reduction in low- density lipoprotein (LDL-C) in select populations reduces risk of atherosclerotic cardiovascular disease (ASCVD) events in both primary and secondary prevention.
  • 19. Current Major Lipid Treatment guidelines 1. 2014 American College of Cardiology/American Health Association (ACC/AHA),4 2. 2014 U.S. Veterans Affairs/Department of Defense (VA-DoD).9 3. 2016 Canadian Cardiovascular Society (CCS),5 4. 2016 European Society of Cardiology/European Atherosclerosis Society (ESC/EAS),6 5. 2016 US Preventive Services Task Force (USPSTF),7,8 6. 2017, American Association of Clinical Endocrinologist (AACE) Examining differences between dyslipidemia guidelines can help identify areas requiring further research and also potential opportunities for harmonization of guidelines.
  • 20. Lipid guidelines • Previous ATP III (Adult Treatment Panel III) looked at LDL goals in combination with patient’s risk. • Current ACC/AHA does not focus on pure lab values, but on overall atherosclerotic cardiovascular risk disease
  • 21. Who should be tested ?? • USPSTF recommends screening: – Men: >35yo – Women: >45yo at increased increased risk – Increased risk is defined as having: diabetes, CHD, family history of CHD in males <50yo and in females <60yo, smokers, HTN, obesity • All patients aged 40-79 years old should have their 10year risk for ASCVD using Pooled Cohort Equation • Of note, European Federation of Clinical Chemistry and Laboratory Medicine recommended in April 2016 non- fasting measurements in most patients.
  • 22. Risk Calculator ASCVD risk <5% = LOW RISK ASCVD 5- 7.5% = INTERMEDIATE ASCVD >7.5% = HIGH RISK http://clincalc.com/Cardiology/ASCVD/PooledCohort.aspx
  • 26. Who needs treatment? • Group1: Clinical ASCVD – ACS, history of MI, angina, stroke, TIA, PAD • Group2: LDL >190 – Rule out secondary causes • Group 3: All Diabetics without clinical ASCVD – 40-75yo with LDL 70-189mg/dL • Group 4: 10 year ASCVD risk >7.5% – with LDL 70-189mg/dL
  • 27. Management • Start statin – Select appropriate dose for patient – Keep potential side effects and drug interactions in mind – If high/moderate intensity not tolerate, use max dose tolerated
  • 28. • Intensity of Statin: – Established ASCVD • Age > 75 moderate intensity statin • Age <75 high intensity statin – LDL cholesterol >190mg/dL • High intensity statin – Diabetes 40-75yo w/LDL 70-189mg/dL and no ASCVD • ASCVD 10 year risk >7.5% high intensity statin • ASCVD 10 year risk < 7.5% moderate intensity statin – No ASCVD or DM, but 10 year ASCVD > 7.5% – Moderate to high intensity statin – Adults >76 yo w/o history of ASCVD • Insufficient evidence to recommend for/against statins
  • 32. The risk of ASCVD and ASCVD-related mortality is substantially greater in the presence of multiple risk factors. Since epidemiologic evidence indicates that ASCVD risk factors frequently cluster, it should be expected that many individuals have multiple risk factors. Recommendations associated with this question: What are the risk factors for ASCVD ? R1. Identify risk factors that enable personalized and optimal therapy for dyslipidemia. (Grade A; BEL 1). Abbreviation: ASCVD, atherosclerotic cardiovascular disease. Jellinger P, Handelsman Y, Rosenblit P, et al. Endocr Practice. 2017;23(4):479-497
  • 33. Major Atherosclerotic Cardiovascular Disease Risk Factors Abbreviations: apo, apolipoprotein; ASCVD, atherosclerotic cardiovascular disease; HDL-C, high-density lipoprotein cholesterol; hsCRP, highly sensitive C-reactive protein; LDL, low-density lipoprotein; LDL-C, low-density lipoprotein cholesterol; Lp-PLA2, lipoprotein-associated phospholipase; PCOS, polycystic ovary syndrome. AACE POSWC. Endocr Pract. 2005;11:126-134; ADA. Diabetes Care. 2017;40(Suppl 1):S1-S135; Brunzell JD, et al. Diabetes Care.2008;31:811-822; Cromwell WC, et al. J Clin Lipidol. 2007;1:583-592; MAJOR RISK FACTORS ADDITIONAL RISK FACTORS NON TRADITIONAL RISK FACTORS Advancing age Increased Total Serum cholesterol level Increased Non HDL – C Increased LDL – C Low HDL –C Diabetes Mellitus Hypertension Stage 3 or 4 chronic kidney disease Cigarette smoking Family history of ASCVD Obesity , abdominal obesity Family history of hyperlipidemia Increased small , dense LDL-C Increased APO – B Increased LDL particle concentration Fasting / Postprandial Hypertriglyceridemia PCOS Increased Lipoprotein (a) Increased Clotting factors Increased inflammation markers (hs- CRP , LP- PLA2) Increased homocysteine Apo E4 isoform Increased uric acid Increased Tgrich remnants
  • 34. How can be the risk assessed? R4. The 10-year risk of a coronary event (high, intermediate, or low) should be determined by detailed assessment using one or more of the following tools (Grade C; BEL 4, upgraded due to cost-effectiveness): • Framingham Risk Assessment Tool • MESA 10-year ASCVD Risk with Coronary Artery Calcification Calculator • Reynolds Risk Score, which includes hsCRP and family history of premature ASCVD • UKPDS risk engine to calculate ASCVD risk in individuals with T2DM R7. When the HDL-C concentration is greater than 60 mg/dL, one risk factor should be subtracted from an individual’s overall risk profile (Grade B; BEL 2). • R8. A classification of elevated TG should be incorporated into risk assessments to aid in treatment decisions (Grade B; BEL 2). Abbreviations: ASCVD, atherosclerotic cardiovascular disease; HDL-C, high-density lipoprotein cholesterol; hsCRP, high-sensitivity CRP; MESA, Multi-Ethnic Study of Atherosclerosis; T2DM, type 2 diabetes mellitus; TG, triglycerides Jellinger P, Handelsman Y, Rosenblit P, et al. Endocr Practice. 2017;23(4):479-497.
  • 36. MESA 10-year ASCVD Risk with Coronary Artery Calcification Calculator
  • 37. Reynolds Risk Score, which includes hsCRP and family history of premature ASCVD
  • 38. UKPDS risk engine to calculate ASCVD risk in individuals with T2DM
  • 39. ASCVD Risk Categories • Low risk: – No risk factors • Moderate risk: – 2 or fewer risk factors and a calculated 10- year risk of less than 10% • High risk: – An ASCVD equivalent including diabetes or stage 3 or 4 CKD with no other risk factors, or individuals with 2 or more risk factors and a 10-year risk of 10%-20% • Very high risk: – Established or recent hospitalization for ACS; coronary, carotid or peripheral vascular disease; diabetes or stage 3 or 4 CKD with 1 or more risk factors; a calculated 10-year risk greater than 20%; or HeFH • Extreme risk: – Progressive ASCVD, including unstable angina that persists after achieving an LDL-C less than 70 mg/dL, or established clinical ASCVD with diabetes, stage 3 or 4 CKD, and/or HeFH, or in those with a history of premature ASCVD (<55 years of age for males or <65 years of age for females) – This category was added in this CPG based on clinical trial evidence and supported by meta-analyses that further lowering of LDL-C produces better outcomes in individuals with ACS. IMPROVE-IT demonstrated lower rates of cardiovascular events in those with ACS when LDL-C levels were lowered to 53 mg/dL combining ezetimibe with statins. Abbreviations: ACS, acute coronary syndrome; ASCVD, atherosclerotic cardiovascular disease; CKD, chronic kidney disease; CPG, clinical practice guideline; HeFH, heterozygous familial hypercholesterolemia; LDL-C, low-density lipoprotein cholesterol; IMPROVE-IT, Improved Reduction of Outcomes: Vytorin Efficacy International Trial. AACE/ACE CPG. 2017;epub ahead of print; Cannon, CP, et al. N Engl J Med. 2015;372(25):2387-239; Jellinger P, Handelsman Y, Rosenblit P, et al. Endocr Practice. 2017;23(4):479-497.
  • 40. Who should be screened and when ? Familial Hypercholesterolemia • R9. Individuals should be screened for FH when there is a family history of: • Premature ASCVD (definite MI or sudden death before age 55 years in father or other male first-degree relative or before age 65 years in mother or other female first-degree relative) or • Elevated cholesterol levels (total, non-HDL, and/or LDL) consistent with FH (Grade C; BEL 4, upgraded due to cost-effectiveness). Adults With Diabetes • R10. Annually screen all adult individuals with T1DM or T2DM for dyslipidemia (Grade B; BEL 2). Jellinger P, Handelsman Y, Rosenblit P, et al. Endocr Practice. 2017;23(4):479- 497
  • 41. Who should be screened and when ? Young Adults (Men Aged 20-45 Years, Women Aged 20-55 Years) • R11. Evaluate all adults 20 years of age or older for dyslipidemia every 5 years as part of a global risk assessment (Grade C; BEL 4, upgraded due to cost-effectiveness). Middle-Aged Adults (Men Aged 45-65 Years, Women Aged 55-65 Years) • R12. In the absence of ASCVD risk factors, screen middle-aged individuals for dyslipidemia at least once every 1 to 2 years. More frequent lipid testing is recommended when multiple global ASCVD risk factors are present (Grade A; BEL 1). • R13. The frequency of lipid testing should be based on individual clinical circumstances and the clinician’s best judgment (Grade C; BEL 4, upgraded due to cost-effectiveness). Jellinger P, Handelsman Y, Rosenblit P, et al. Endocr Practice. 2017;23(4):479-497
  • 42. • R14. Annually screen older adults with 0 to 1 ASCVD risk factor for dyslipidemia (Grade A; BEL 1). • R15. Older adults should undergo lipid assessment if they have multiple ASCVD global risk factors (i.e., other than age) (Grade C; BEL 4, upgraded due to cost-effectiveness). • R16. Screening for this group is based on age and risk, but not gender; therefore, older women should be screened in the same way as older men (Grade A; BEL 1). Older Adults (Older Than 65 Years)
  • 43. Whom to screen ? *Men younger than 55 years and women younger than 65 years of age in first-degree relative. BMI, body mass index.
  • 44. How to screen ? ApoB, apolipoprotein B; eGFR, estimated glomerular filtration rate; HDL-C,high-density lipoprotein cholesterol; LDL-C, low- density lipoprotein cholesterol; TC, total cholesterol; TG, triglycerides
  • 45. Familial Hypercholesterolemia: Diagnosis • FH diagnostic criteria include lipid levels and family history, physical symptoms (if any), and genetic analysis (if available)1 • Three clinical diagnostic tools:2-3 • Simon Broome Register Diagnostic Criteria • Dutch Lipid Clinic Network Diagnostic Criteria • U.S. MEDPED • Factors that lead to an FH diagnosis include: • Premature ASCVD, fasting LDL-C >190 mg/dL, the presence of tendon xanthomas, full corneal arcus in individuals <40 years of age, or a family history of high cholesterol and/or premature ASCVD1 • While genetic testing may identify FH, it is not commonly used in the United States due to cost and lack of payer coverage1 Abbreviations: ASCVD, atherosclerotic cardiovascular disease; FH, familial hypercholesterolemia; LDL-C, low-density lipoprotein cholesterol; MEDPED, Make Early Diagnoses Prevent Early Deaths Program Diagnostic Criteria. 1. Bouhairie VE, et al. Cardiol Clin. 2015;33:169-179; 2. Haralambos K, et al. Curr Opin Lipidol. 2016;27:367-374; 3. Turgeon RD, et al. Can Fam Physician. 2016;62:32-37.
  • 46. Familial Hypercholesterolemia: Prevalence and Risk • FH is caused by genetic mutations passed on by: • One parent (heterozygous, HeFH)1 • Both parents (homozygous, HoFH)1 • HoFH prevalence ranges from 1 in 160,000 to 1 in 250,0002,3 • Individuals with HoFH have extremely high LDL-C levels (>500 mg/dL) and premature CV risk4 • Many with HoFH experience their first coronary event in childhood or adolescence4 • HeFH prevalence ranges from 1 in 200 to 1 in 2503 • Individuals with HeFH can present with LDL-C levels 90 to 500 mg/dL and have premature CV risk4 • On average, individuals with HeFH experience their first coronary event at age 42 (about 20 years younger than the general population)4 • Early treatment is recommended for all individuals with FH, with a goal of reducing LDL-C levels by 50% from baseline3 Abbreviations: CV, cerebrovascular; FH, familial hypercholesterolemia; HeFH, heterozygous familial hypercholesterolemia; HoFH, homozygous familial hypercholesterolemia; LDL-C, low-density lipoprotein cholesterol. 1. Zimmerman MP. Am Health Drug Benefits. 2015;8:436-442; 2. Goldstein J, et al. The Metabolic and Molecular Bases of Inherited Disease. 7th ed. New York, NY: McGraw-Hill; 1995: 1981-2030; 3. Bouhairie VE, et al. Cardiol Clin. 2015;33:169-179; 4. Turgeon RD, et al. Can Fam Physician. 2016;62:32-37.
  • 47. Which screening tests should be used? Fasting Lipid Profile • R19. Use a fasting lipid profile to ensure the most precise lipid assessment; this should include total cholesterol, LDL-C, TG, and non-HDL-C (Grade C; BEL 4, upgraded due to cost- effectiveness). • R20. Lipids, including TG, can be measured in the non-fasting state if fasting determinations are impractical (Grade D). LDL-C • R21. LDL-C may be estimated using the Friedewald equation: LDL-C = (total cholesterol – HDL-C) – TG/5; however, this method is valid only for values obtained during the fasting state and becomes increasingly inaccurate when TG levels are greater than 200 mg/dL, and becomes invalid when TG levels are greater than 400 mg/dL (Grade C; BEL 3). • R22. LDL-C should be directly measured in certain high-risk individuals, such as those with fasting TG levels greater than 250 mg/dL or those with diabetes or known vascular disease (Grade C; BEL 3). HDL-C • R23. Measurement of HDL-C should be included in screening tests for dyslipidemia (Grade B; BEL 2). Abbreviations: HDL-C, high-density lipoprotein cholesterol; LDL-C, low-density lipoprotein cholesterol; TG, triglycerides. Jellinger P, Handelsman Y, Rosenblit P, et al. Endocr Practice. 2017;23(4):479-497.
  • 48. Which screening tests should be used? Non-HDL-C • R24. Non-HDL-C (total cholesterol minus HDL-C) should be calculated to assist risk stratification in individuals with moderately elevated TG (200 to 500 mg/dL), diabetes, and/or established ASCVD (Grade B; BEL 2). • R25. If insulin resistance is suspected, non-HDL-C should be evaluated to gain useful information regarding the individual’s total atherogenic lipoprotein burden (Grade D). Triglycerides • R26. TG levels should be part of routine lipid screening: moderate elevations (≥150 mg/dL) may identify individuals at risk for insulin resistance syndrome and levels ≥200 mg/dL may identify individuals at substantially increased ASCVD risk (Grade B; BEL 2). Apolipoproteins • R27. Apo B and/or an apo B/apo A1 ratio calculation and evaluation may be useful in at-risk individuals (TG ≥150, HDL-C <40, prior ASCVD event, T2DM, and/or insulin resistance syndrome [even at target LDL-C levels]) to assess residual risk and guide decision-making (Grade A; BEL 1). • R28. Apo B measurements (reflecting the particle concentration of LDL and all other atherogenic lipoproteins) may be useful to assess the success of LDL-C–lowering therapy (Grade A; BEL 1). Abbreviations: apo, apolipoprotein; ASCVD, atherosclerotic cardiovascular disease; HDL-C, high-density lipoprotein cholesterol; LDL-C, low- density lipoprotein cholesterol; T2DM, type 2 diabetes mellitus; TG, triglycerides. Jellinger P, Handelsman Y, Rosenblit P, et al. Endocr Practice. 2017;23(4):479-497
  • 49. Additional screening test Coronary artery calcification • R33. Coronary artery calcification measurement has been shown to be of high predictive value and is useful in refining risk stratification to determine the need for more aggressive treatment strategies (Grade B; BEL 2). hsCRP • R30. Use hsCRP to stratify ASCVD risk in individuals with a standard risk assessment that is borderline, or in those with an intermediate or higher risk with an LDL-C concentration less than 130 mg/dL (Grade B; BEL 2). Lp-PLA2 • R31. Measure lipoprotein- associated phospholipase A2 (Lp-PLA2), which in some studies has demonstrated more specificity than hsCRP, when it is necessary to further stratify an individual’s ASCVD risk, especially in the presence of hsCRP elevations (Grade A; BEL 1). Homocysteine • R32. The routine measurement of homocysteine, uric acid, plasminogen activator inhibitor-1, or other inflammatory markers is not recommended because the benefit of doing so is not sufficiently proven (Grade D). Carotid intima media thickness • R34. Carotid intima media thickness may be considered to refine risk stratification to determine the need for more aggressive ASCVD preventive strategies (Grade B; BEL 2). Abbreviations: apo, apolipoprotein; ASCVD, atherosclerotic cardiovascular disease; HDL-C, high-density lipoprotein cholesterol; LDL-C, low-density lipoprotein cholesterol; T2DM, type 2 diabetes mellitus; TG, triglycerides. Jellinger P, Handelsman Y, Rosenblit P, et al. Endocr Practice. 2017;23(4):479-497
  • 50. •R5. Special attention should be given to assessing women for ASCVD risk by determining the 10-year risk (high, intermediate, or low) of a coronary event using the Reynolds Risk Score or the Framingham (Grade C; BEL 4, upgraded due to cost-effectiveness). Risk Assessment •R72. Women should be evaluated for their ASCVD risk and be treated with pharmacotherapy if lifestyle intervention is insufficient (Grade C; BEL 4; upgraded due to potential benefit). •R73. Hormone replacement therapy for the treatment of dyslipidemia in postmenopausal women is not recommended (Grade A; BEL 1). •An HDL-C concentration <40 mg/dL is an established independent risk factor for ASCVD in both men and women. However, because HDL-C levels tend to be higher in women than in men, an HDL-C concentration <50 mg/dL in women is also considered a marginal risk factor. •In stark contrast to findings in men, very low HDL-C (<40 mg/dL) is an independent risk factor for ASCVD development and mortality in women, even in the presence of total cholesterol concentrations less than 200 mg/dL or normal LDL-C and/or TG levels. Compared with women with high HDL-C, women with low HDL-C have a nearly 3-fold elevated risk of ASCVD. Treatment options What special consideration should be given for women ? Jellinger P, Handelsman Y, Rosenblit P, et al. Endocr Practice. 2017;23(4):479-497
  • 51. Common Secondary Causes of Dyslipidemia
  • 52. What are lipid treatment goals? R35. Treatment goals for dyslipidemia should be personalized according to levels of risk (Grade A; BEL 1). R36. For individuals at low risk (i.e., with no risk factors), an LDL-C goal of less than 130 mg/dL is recommended (Grade A; BEL 1). R37. For individuals at moderate risk (i.e., with 2 or fewer risk factors and a calculated 10-year risk of less than 10%), an LDL-C goal of less than 100 mg/dL is recommended (Grade A; BEL 1). R38. For individuals at high risk (i.e., with an ASCVD equivalent including diabetes or stage 3 or 4 CKD with no other risk factors, or individuals with 2 or more risk factors and a 10-year risk of 10%-20%), an LDL-C goal of less than 100 mg/dL is recommended (Grade A; BEL 1). Jellinger P, Handelsman Y, Rosenblit P, et al. Endocr Practice. 2017;23(4):479-497.
  • 53. R39. For individuals at very high risk (i.e., with established or recent hospitalization for ACS; coronary, carotid or peripheral vascular disease; diabetes or stage 3 or 4 CKD with 1 or more risk factors; a calculated 10-year risk greater than 20%; or HeFH), an LDL-C goal of less than 70 mg/dL is recommended (Grade A; BEL 1). R40. For individuals at extreme risk (i.e., with progressive ASCVD, including unstable angina that persists after achieving an LDL-C less than 70 mg/dL, or established clinical ASCVD in individuals with diabetes, stage 3 or 4 CKD, and/or HeFH, or in individuals with a history of premature ASCVD (<55 years of age for males or <65 years of age for females), an LDL-C goal of less than 55 mg/dL is recommended (Grade A; BEL 1). R41. An LDL-C goal of <100 mg/dL is considered “acceptable” for children and adolescents, with 100 to 129 mg/dL considered “borderline” and 130 mg/dL or greater considered “high” (based on recommendations from the American Academy of Pediatrics) (Grade D). Jellinger P, Handelsman Y, Rosenblit P, et al. Endocr Practice. 2017;23(4):479-497.
  • 54. High-Density Lipoprotein Cholesterol •R42. HDL-C should be greater than 40 mg/dL, but also as high as possible, primarily through the use of lifestyle interventions (e.g., weight loss, physical activity, and tobacco cessation), and if risk factors are present (e.g., borderline elevated LDL-C levels, a family history of premature ASCVD, or a personal history of ASCVD), also through the use of pharmacotherapy primarily focused on reducing LDL-C (Grade A; BEL 1). Non–High-Density Lipoprotein Cholesterol •R43. For most individuals, a non–HDL-C goal (total cholesterol minus HDL-C) 30 mg/dL higher than the individual’s specific LDL-C goal is recommended (Grade D). •R44. For individuals at extreme risk, a non-HDL-C goal 25 mg/dL higher than the individual-specific LDL-C goal is recommended (Grade A; BEL 1). Apolipoproteins •R45. For individuals at increased risk of ASCVD, including those with diabetes, an optimal apo B goal is less than 90 mg/dL, while for individuals with established ASCVD or diabetes plus 1 or more additional risk factor(s), an optimal apo B goal is less than 80 mg/dL, and for individuals at extreme risk, an optimal apo B goal is less than 70 mg/dL (Grade A; BEL 1). Triglycerides •R46. TG goals of less than 150mg/dL are recommended (Grade A; BEL 1).
  • 55. Treatment goals AACE/ACE 2017;epub ahead of print; Baigent C, et al. Lancet. 2010;376:1670-1681; Boekholdt SM, et al. J Am Coll Cardiol. 2014;64(5):485-494; Brunzell JD, et al. Diabetes Care. 2008;31:811-822; Cannon CP, et al. N Engl J Med. 2015;372(25):2387-2397; Heart Protection Study Collaborative Group. Lancet. 2002;360:7-22; Jellinger P, Handelsman Y, Rosenblit P, et al. Endocr Practice. 2017;23(4):479-497; Ridker PM, J Am Coll Cardiol. 2005;45:1644-1648; Sever PS, et al. Lancet. 2003;361:1149-1158; Shepherd J, et al. Lancet. 2002;360:1623-1630; Weiner DE, et al. J Am Soc Nephrol. 2004;15(5):1307-1315.
  • 56. TREATMENT GOALS Barter PJ, et al. J Intern Med. 2006;259:247-258; Boekholdt SM, et al. J Am Coll Cardiol. 2014;64(5):485-494; Brunzell JD, et al. Diabetes Care. 2008;31:811-822; Cannon CP, et al. N Engl J Med. 2015;372(25):2387-2397; Grundy SM, et al. Circulation. 2004;110:227-239; Heart Protection Study Collaborative Group. Lancet. 2002;360:7-22; Jellinger P, Handelsman Y, Rosenblit P, et al. Endocr Practice. 2017;23(4):479-497; Lloyd-Jones DM, et al. Am J Cardiol. 2004;94:20-24; McClelland RL, et al. J Am Coll Cardiol. 2015;66(15):1643-1653; NHLBI. NIH Publication No. 02-5215. 2002; Ridker PM, J Am Coll Cardiol. 2005;45:1644-1648; Ridker PM, et al. JAMA. 2007;297(6):611-619; Sever PS, et al. Lancet. 2003;361:1149-1158; Shepherd J, et al. Lancet. 2002;360:1623-1630; Smith SC Jr, et al. Circulation. 2006;113:2363-2372; Stevens RJ, et al. Clin Sci. 2001;101(6):671-679; Stone NJ. Am J Med. 1996;101:4A40S-48S; Weiner DE, et al. J Am Soc Nephrol. 2004;15(5):1307-1315.
  • 57. CLASSIFICATION OF ELEVATED TRIGLYCERIDE LEVELS Einhorn D, et al. Endocr Pract. 2003;9:237-252; Frick MH, et al. NEJM. 1987;317:1237-1245; Jellinger P, Handelsman Y, Rosenblit P, et al. Endocr Practice. 2017;23(4):479-497; Keech A, et al. Lancet. 2005;366:1849- 1861; NHLBI. NIH Publication No. 02-5215. 2002; Tenaknen L, et al. Arch Intern Med. 2006;166:743-748. TG levels that are even moderately elevated (≥150 mg/dL) may identify individuals at risk for the insulin resistance syndrome. TG levels ≥200 mg/dL may indicate a substantial increase in ASCVD risk. Hypertriglyceridemia is also commonly associated with a procoagulant state and hypertension.
  • 58. What treatments are available for dyslipidemia? Treatment categories for dyslipidemia: – Lifestyle changes • Physical activity • Medical nutrition therapy • Smoking cessation – Pharmacologic therapy • Statins • Fibrates • Omega-3 fish oil • Niacin • Bile acid sequestrants • Cholesterol absorption inhibitors • PCSK9 inhibitors • MTP inhibitor • Antisense apo B oligonucleotide • Combination therapies Abbreviations: apo, apolipoprotein; MTP, microsomal transfer protein; PCSK9, proprotein convertase subtilisin/kexin type 9.
  • 59. Non pharmacological lipid treatment Physical Activity • R48. A reasonable and feasible approach to fitness therapy (i.e., exercise programs that include at least 30 minutes of moderate-intensity physical activity [consuming 4-7 kcal/min] 4 to 6 times weekly, with an expenditure of at least 200 kcal/day) is recommended; suggested activities include brisk walking, riding a stationary bike, water aerobics, cleaning/scrubbing, mowing the lawn, and sporting activities (Grade A; BEL 1). • R49. Daily physical activity goals can be met in a single session or in multiple sessions throughout the course of a day (10 minutes minimum per session); for some individuals, breaking activity up throughout the day may help improve adherence with physical activity programs (Grade A; BEL 1). • R50. In addition to aerobic activity, muscle-strengthening activity is recommended at least 2 days a week (Grade A; BEL 1). Medical Nutrition Therapy • R51. For adults, a reduced-calorie diet consisting of fruits and vegetables (combined ≥5 servings/day), grains (primarily whole grains), fish, and lean meats is recommended (Grade A; BEL 1). • R52. For adults, the intake of saturated fats, trans-fats, and cholesterol should be limited, while LDL-C-lowering macronutrient intake should include plant stanols/sterols (~2 g/ day) and soluble fiber (10-25 g/day) (Grade A; BEL 1). • R53. Primary preventive nutrition consisting of healthy lifestyle habits is recommended in all healthy children (Grade A; BEL 1). Smoking Cessation • R54. Tobacco cessation should be strongly encouraged and facilitated (Grade A; BEL 2; upgraded due to potential benefit). Jellinger P, Handelsman Y, Rosenblit P, et al. Endocr Practice. 2017;23(4):479-497
  • 60. Statins, Fibrates Statins • R56. Statin therapy is recommended as the primary pharmacologic agent to achieve target LDL-C goals on the basis of morbidity and mortality outcome trials (Grade A; BEL 1). • R57. For clinical decision making, mild elevations in blood glucose levels and/or an increased risk of new-onset T2DM associated with intensive statin therapy do not outweigh the benefits of statin therapy for ASCVD risk reduction (Grade A, BEL 1). • R58. In individuals within high-risk and very high-risk categories, further lowering of LDL-C beyond established targets with statins results in additional ASCVD event reduction and may be considered (Grade A, BEL 1). • R59. Very high-risk individuals with established coronary, carotid, and peripheral vascular disease, or diabetes, who also have at least 1 additional risk factor, should be treated with statins to target a reduced LDL-C treatment goal of <70 mg/dL (Grade A, BEL 1). • R60. Extreme risk individuals should be treated with statins or with combination therapy to target an even lower LDL-C treatment goal of <55 mg/dL (Grade A, BEL 1). Fibrates • R61. Fibrates should be used to treat severe hypertriglyceridemia (TG >500 mg/dL) (Grade A; BEL 1). • R62. Fibrates may improve ASCVD outcomes in primary and secondary prevention when TG concentrations are 200 mg/dL and HDL-C concentrations <40 mg/dL (Grade A; BEL 1). Jellinger P, Handelsman Y, Rosenblit P, et al. Endocr Practice. 2017;23(4):479-497
  • 61. Statin Main Considerations • Liver function test prior to therapy and as clinically indicated thereafter • Myalgias and muscle weakness in some individuals • Potential for drug-drug interaction between some statins and CYP450 3A4 inhibitors, cyclosporine, warfarin, and protease inhibitors • Myopathy/rhabdomyolysis in rare cases; increased risk with coadministration of some drugs (see product labeling) • Simvastatin dosages should not exceed 40 mg in most individuals; dosages of 80 mg are no longer recommended except in those who have tolerated 80 mg for 12 months or more without muscle toxicity • Do not exceed 20 mg simvastatin daily with amlodipine or ranolazine • Plasma elevations of rosuvastatin may be higher among Asian persons than other ethnic groups • New-onset diabetes is increased in individuals treated with statins; however, it is dose- related, occurs primarily in individuals with MetS, appears to be less common with pravastatin and possibly pitavastatin, and occurs overall to a lesser extent than the associated decrease in ASCVD Metabolic Effects • Primarily ↓ LDL-C 21%-55% by competitively inhibiting rate-limiting step of cholesterol synthesis in the liver, leading to upregulation of hepatic LDL receptors • Effects on TG and HDL-C are less pronounced (↓ TG 6%-30% and ↑ HDL-C 2%-10%) Bissonnette S, et al. Can J Cardiol. 2006;22:1035-1044; Denke M, et al. Diab Vasc Dis Res. 2006;3:93-102; Jellinger P, Handelsman Y, Rosenblit P, et al. Endocr Practice. 2017;23(4):479-497; Preiss D, et al. JAMA. 2011;305: 2556-2564
  • 62. Statin and dosages Agent Usual recommended starting daily dosage Dosage range Method of administration Statins Lovastatin 20 mg 10-80 mg Oral Pravastatin 40 mg 10-80 mg Oral Simvastatin 20-40 mg 5-80 mga Oral Fluvastatin 40 mg 20-80 mg Oral Atorvastatin 10-20 mg 10-80 mg Oral Rosuvastatin 10 mg 5-40 mg Oral Pitavastatin 2 mg 2-4 mg Oral Simvastatin, 80 mg, not approved for therapy unless individual has been on treatment for more than 1 year without myopathy. Crestor (rosuvastatin calcium); [PI]; 2016; Jellinger P, Handelsman Y, Rosenblit P, et al. Endocr Practice. 2017;23(4):479-497; Lescol (fluvastatin sodium) [PI]; 2012 Lipitor (atorvastatin calcium) [PI]; 2015; Livalo (pitavastatin) [PI]; 2013; ; Mevacor (lovastatin) [PI]; 2014; Pravachol (pravastatin sodium) [PI]; 2016; Zocor (simvastatin) [PI]; 2015.
  • 63. Fibrates Metabolic Effects: • Primarily ↓ TG 20%-35%, ↑ HDL-C 6%-18% by stimulating lipoprotein lipase activity • Fenofibrate may ↓ TC and LDL-C 20%-25% • Lower VLDL-C and LDL-C; reciprocal rise in LDL-C transforms the profile into a less atherogenic form by shifting fewer LDL particles to larger size • Fenofibrate ↓ fibrinogen level Abbreviations: HDL-C, high-density lipoprotein cholesterol; LDL, low-density lipoprotein, LDL-C, low-density lipoprotein cholesterol; TC, total cholesterol; TG, triglycerides; VLDL-C, very low-density lipoprotein cholesterol. Aguilar-Salinas CA, et al. Metabolism. 2001;50:729-733; Athyros VG, et al. Coron Artery Dis. 1995;6:25-1256; Avellone G, et al. Blood Coagul Fibrinolysis. 1995;6:543-548; Bröijersen A, et al. Arterioscler Thromb Vasc Biol. 1996;16:511-516; Bröijersén A, et al. Thromb Haemost. 1996;76:171-176; Davidson MH, et al. Am J Cardiol. 2007;99:3C-18C; Farnier M, et al. Eur Heart J. 2005;26:897-905; Guyton JR, et al. Arch Intern Med. 2000;160:1177-1184; Hottelart C, et al. Nephron. 2002;92:536-541; Insua A, et al. Endocr Pract. 2002;8:96-101; Jellinger P, Handelsman Y, Rosenblit P, et al. Endocr Practice. 2017;23(4):479-497; Kockx M, et al. Thromb Haemost. 1997;78:1167-1172; Lopid (gemfibrozil) [PI] 2010; McKenney JM, et al. J Am Coll Cardiol. 2006;47:1584-1587; Syvänne M, et al. Atherosclerosis. 2004;172:267-272; Tricor (fenofibrate) [PI]; 2010; Trilipix (fenofibric acid) [PI]; 2016; Westphal S, et al. Lancet. 2001; 358:39-40.
  • 64. Fibrates metabolic effects and main consideration Main Considerations: Gemfibrozil may ↑ LDL-C 10%-15% GI symptoms, possible cholelithiasis May potentiate effects of orally administered anticoagulants Gemfibrozil may ↑ fibrinogen level Gemfibrozil and fenofibrate can ↑ homocysteine independent of vitamin concentrations May cause muscle disorders; myopathy/rhabdomyolysis when used with statin Fibrates are associated with increased serum creatinine levels, which may not reflect renal dysfunction Fenofibrate dose should be cut by two-thirds and gemofibrozil by one-half when eGFR is 15-60, and fibrates should be avoided when eGFR is <15 Can improve diabetic retinopathy
  • 65. Omega 3 fish oil Metabolic Effects: • ↓ TG 27%-45%, TC 7%-10%, VLDL-C 20%-42%, apo B 4%, and non-HDL-C 8%-14% in individuals with severe hypertriglyceridemia most likely by reducing hepatic VLDL-TG synthesis and/or secretion and enhancing TG clearance from circulating VLDL particles. Other potential mechanisms of action include: increased ß- oxidation; inhibition of acyl-CoA; 1,2-diacylglyceral acyltransferase; decreased hepatic lipogenesis; and increased plasma lipoprotein activity. • Icosapent ethyl ↓ LDL-C 5%, whereas, omega-3-acid ethyl esters ↑ LDL-C 45%
  • 66. Omega 3 fatty aids Assess TG levels prior to initiating and periodically during therapy. Omega-3-acid ethyl esters can increase LDL-C levels. Monitor LDL-C levels during treatment. May prolong bleeding time. Monitor coagulation status periodically in patients receiving treatment with omega-3 fatty acids and other drugs affecting coagulation. Monitor ALT and AST levels periodically during treatment in patients with hepatic impairment. Some patients may experience increases in ALT levels only. Exercise caution when treating patients with a known hypersensitivity to fish and/or shellfish.
  • 67. Omega 3 fattyacids Assess TG levels prior to initiating and periodically during therapy. Omega-3-acid ethyl esters can increase LDL-C levels. Monitor LDL-C levels during treatment. May prolong bleeding time. Monitor coagulation status periodically in patients receiving treatment with omega-3 fatty acids and other drugs affecting coagulation. Monitor ALT and AST levels periodically during treatment in patients with hepatic impairment. Some patients may experience increases in ALT levels only. Exercise caution when treating patients with a known hypersensitivity to fish and/or shellfish.
  • 68. Agent Usual recommended starting daily dosage Dosage range Method of administration Bile acid sequestrants Cholestyramine 8-16 g 4-24 g Oral Colestipol 2 g 2-16 g Oral Colesevelam 3.8 g 3.8-4.5 g Oral Metabolic Effects: • Primarily ↓ LDL-C 15%-25% by binding bile acids and preventing their reabsorption in the ileum (causing hepatic cholesterol depletion and LDL- receptor upregulation) • Colesevelam ↓ glucose and hemoglobin A1C (~0.5%); FDA-approved to treat T2DM Abbreviations: A1C, glycated hemoglobin; FDA, Food and Drug Administration; LDL, low-density lipoprotein; LDL-C, low- density lipoprotein cholesterol; T2DM, type 2 diabetes mellitus; TG, triglyceride. Colestid (colestipol hydrochloride) [PI]; 2014; Jellinger P, Handelsman Y, Rosenblit P, et al. Endocr Practice. 2017;23(4):479- 497; Prevalite (cholestyramine for oral suspension, USP) [PI]; 2015; WelChol (colesevelam hydrochloride) [PI]; 2014; Zieve FJ, et al. Ther. 2007;29:74-839:74-83.
  • 69. Bile acid sequestrants • Main Considerations: •May ↑ serum TG •Frequent constipation and/or bloating, which can reduce adherence •Many potential drug interactions (decreased drug absorption), less so with colesevelam (see product labeling) •May reduce absorption of folic acid and fat- soluble vitamins such as vitamins A, D, and K
  • 70. Metabolic Effects • Primarily ↓ LDL-C 10%-18% by inhibiting intestinal absorption of cholesterol and decreasing delivery to the liver, leading to upregulation of hepatic LDL receptors • ↓ Apo B 11%-16% • In combination with statins, additional ↓ LDL-C 25%, total ↓ LDL-C 34%-61% • In combination with fenofibrate, ↓ LDL-C 20%-22% and ↓ apo B 25%-26% without reducing ↑ HDL-C Main Considerations • Myopathy/rhabdomyolysis (rare) • When coadministered with statins or fenofibrate, risks associated with those drugs remain (e.g., myopathy/ rhabdomyolysis, cholelithiasis)
  • 71. Niacin • Metabolic Effects: • ↓ LDL-C 10%-25%, ↓ TG 20%-30%, ↑ HDL-C 10%-35% by decreasing hepatic synthesis of LDL-C and VLDL-C • ↓ Lipoprotein (a) • Transforms LDL-C to less atherogenic form by increasing average particle size and also decreases LDL particle concentration Usual recommended starting daily dosage Dosage range Method of administration Niacin (nicotinic acid) Immediate release 250 mg 250-3,000 mg Oral Extended release 500 mg 500-2,000 mg Oral
  • 72. Niacin • Main Considerations: • Potential for frequent skin flushing, pruritus, abdominal discomfort, hepatoxicity (rare but may be severe), nausea, peptic ulcer, atrial fibrillation • Deleterious effect on serum glucose at higher dosages • Increases uric acid levels; may lead to gout
  • 73. Metabolic Effects: • ↓LDL-C 48%-71%, ↓ non-HDL-C 49%-58%, ↓TC 36%-42%, ↓Apo B 42%-55% by inhibiting PCSK9 binding with LDLRs, increasing the number of LDLRs available to clear LDL, and lowering LDL-C levels Jellinger P, Handelsman Y, Rosenblit P, et al. Endocr Practice. 2017;23(4):479- 497; Praluent (alirocumab) [PI] 2015; Repatha (evolocumab) [PI]; 2016.
  • 74. • Main Considerations: • Require subcutaneous self-injection; refrigeration generally needed • Overall levels of adverse reactions and discontinuation very low • Adverse reactions with significantly different rates between drug and placebo were: local injection site reactions and influenza • The most common adverse reactions with similar rates for drug vs. placebo were: • Alirocumab: nasopharyngitis, influenza, urinary tract infections, diarrhea, bronchitis, and myalgia • Evolocumab: nasopharyngitis, back pain, and upper respiratory tract infection
  • 75. MTP Inhibitor Metabolic Effects: • ↓ Up to LDL-C 40%, TC 36%, apo B 39%, TG 45%, and non-HDL-C 40% (depending on dose) in individuals with HoFH by binding and inhibiting MTP, which inhibits synthesis of chylomicrons and VLDL Abbreviations: ALT, aspartate amino transferase; AST, amino alanine transferase; FDA, Food dministration; HDL-C, high-density lipoprotein cholesterol; HoFH, homozygous familial hypercholesterolemia; LDL-C, low-density lipoprotein cholesterol; MTP, microsomal transfer protein; REMS, Risk Evaluation and Mitigation Strategy; TG, triglycerides; VLDL, very low-density lipoprotein. Jellinger P, Handelsman Y, Rosenblit P, et al. Endocr Practice. 2017;23(4):479-497; Juxtapid (lomitapide) [PI]; 2012.
  • 76. MTP Inhibitor • Main Considerations: • Can cause increases in transminases (ALT, AST); monitoring of ALT, AST, alkaline phosphatase, and total bilirubin prior to initiation, and of ALT and AST during treatment, is required per FDA REMS • Causes increases in hepatic fat (steatosis) with or without concomitant elevated transminases, which may be a risk for progressive liver diseases • Also causes steatosis of the small intestine with resulting abdominal pain and steatorrhea unless a very-low-fat diet is followed; may also cause fat-soluble vitamin deficiency unless vitamin supplements are taken • Caution should be exercised when used with other drugs with potential hepatoxicity; because of hepatoxicity risk, only available through REMS program
  • 77. • Metabolic Effects • ↓ LDL-C 21%, TC 19%, apo B 24%, and non-HDL-C 22% in individuals with HoFH by degrading mRNA for apo B-100, the principal apolipoprotein needed for hepatic synthesis of VLDL (and subsequent intra-plasma production of IDL and LDL) Agent Usual recommended starting daily dosage Dosage range Method of administration Anti-sense apolipoprotein B oligonucleotide Mipomersen (SQ injection) 200 mg once weekly 200 mg once weekly SQ
  • 78. mipomersen • Main Considerations: • Can cause increases in transminases (ALT, AST); monitoring of ALT, AST, alkaline phosphatase, and total bilirubin before initiation, and of ALT and AST during treatment is recommended • Causes increases in hepatic fat (steatosis) with or without concomitant elevated transminases, which may be a risk for progressive liver diseases • Caution should be exercised when used with other drugs with potential hepatoxicity; because of hepatoxicity risk, only available through REMS program Jellinger P, Handelsman Y, Rosenblit P, et al. Endocr Practice. 2017;23(4):479-497; Kynamro (mipomersen sodium) Injection [PI]; 2016
  • 79. How should treatment be monitored? • R75. Reassess individuals’ lipid status 6 weeks after therapy initiation and again at 6-week intervals until the treatment goal is achieved (Grade D; BEL 4). R76. While on stable lipid therapy, individuals should be tested at 6- to 12-month intervals (Grade D; BEL 4). R77. While on stable lipid therapy, the specific interval of testing should depend on individual adherence to therapy and lipid profile consistency; if adherence is a concern or the lipid profile is unstable, the individual will probably benefit from more frequent assessment (Grade C; BEL 4; upgraded due to potential benefit). Jellinger P, Handelsman Y, Rosenblit P, et al. Endocr Practice. 2017;23(4):479-497
  • 80. R79. Liver transaminase levels should be measured before and 3 months after niacin or fibric acid treatment initiation because most liver abnormalities occur within 3 months of treatment initiation. Liver transaminase levels should be measured periodically thereafter (e.g., semiannually or annually) (Grade C; BEL 4; upgraded due to potential benefit). R80. Creatine kinase levels should be assessed and the statin discontinued, at least temporarily, when an individual reports clinically significant myalgias or muscle weakness on statin therapy (Grade C; BEL 4; upgraded due to potential benefit). Jellinger P, Handelsman Y, Rosenblit P, et al. Endocr Practice. 2017;23(4):479- 497 R78. More frequent lipid status evaluation is recommended in situations such as deterioration of diabetes control, use of a new drug known to affect lipid levels, progression of atherothrombotic disease, considerable weight gain, unexpected adverse change in any lipid parameter, development of a new ASCVD risk factor, or convincing new clinical trial evidence or guidelines that suggest stricter lipid goals (Grade C; BEL 4; upgraded due to potential benefit).
  • 81. •Lipid Association of India (LAI) expert consensus statement on management of dyslipidemia in Indians 2017: Part 2 SS Iyengar et al. Lipid Association of India (LAI) expert consensus statement on management of dyslipidemia in Indians 2017: part 2, Clinical Lipidology, 12:1, 56-109 Focus on Dyslipidaemia and Comorbidities
  • 82. Objective of the guideline  Lipid Association of India (LAI) part 2 recommendations refer to specific patient populations. These part 2 LAI recommendations focus on comorbidities associated with Dyslipidemia and also focuses on specific patient groups.  These include patients with heart failure, chronic kidney disease, non-alcoholic fatty liver disease, cerebrovascular disease, thyroid disorders, inflammatory joint diseases, familial hypercholesterolemia and human immunodeficiency virus infection.  Guideline also consider women, the elderly and post- transplantation patients.
  • 83. Dyslipidemia in Heart Failure  Besides lifestyle measures, statins may be administered to patients with ischemic HF with NYHA Class II-III symptoms.  Patients with advanced symptomatic HF require individualized care, in these groups, intensive statin therapy with a goal to achieve 50% reduction in LDL-C levels is justified.  Statins are also recommended for ischemic HF patients awaiting heart transplantation. HF: Heart Failure, LDL-C; Low-density Lipoprotein, NYHA: New York Heart Association class
  • 84. Dyslipidemia in Heart Failure  Statins are not recommended in NYHA Class IV HF. However, three points can be considered in patients with advanced HF. Firstly, no excess of side effects. Secondly, the degree of LDL-C lowering did not correlate with event rate. Thirdly, there were fewer hospitalizations  Statin therapy is not recommended in advanced HF patients who have a short life expectancy (e.g. because of comorbidities like malignancy).  Statins are not recommended in non-ischemic HF or dilated cardiomyopathy
  • 85. Dyslipidemia in CKD  Follow-up measurement of lipid levels should be performed routinely (at treating physicians’ discretion) in subjects with CKD until the LDL-C and non- HDL-C target is achieved.  Therapeutic lifestyle modification should be recommended to all patients.  A combination of statins/statin plus ezetimibe is recommended for all adults over the age of 40 years with eGFR < 60 mL/min/1.73 m2.  Adults between the ages of 18 and 39, with CKD should be treated with statins if they have one of the following: – - Known CAD, stroke or peripheral arterial disease – - Diabetes mellitus – - Life time risk of ASCVD >30% Kodachrome Kidney Disease, LDL-C: Low-density Lipoprotein, HDL-C: High-density Lipoprotein, eGFR: Estimated Glomerular Filtration Rate,, CAD: coronary artery disease, ASCVD: Atherosclerotic cardiovascular risk factors
  • 86. Dyslipidemia in CKD  Patients who are already receiving lipid-lowering therapy at the time of starting dialysis should continue to receive these agents.  In adults with dialysis-dependent CKD who are not on statins, statins could be considered at the lowest dose possible and should be titrated up carefully to achieve the LDL-C level for very high risk patients  All adult kidney transplant recipients should receive statin therapy.  Statins should be started at a lower dose in all subjects with eGFR <30 ml/min with gradual escalation to the recommended dose along with monitoring for any adverse events Kodachrome Kidney Disease, LDL-C: Low-density Lipoprotein, HDL-C: High-density Lipoprotein, eGFR: Estimated Glomerular Filtration Rate,, CAD: coronary artery disease, ASCVD: Atherosclerotic cardiovascular
  • 87. Dyslipidemia in CKD • Renal dosing for commonly available statins Kodachrome Kidney Disease, LDL-C: Low-density Lipoprotein, HDL- C: High-density Lipoprotein, eGFR: Estimated Glomerular Filtration Rate,, CAD: coronary artery disease, ASCVD: Atherosclerotic cardiovascular risk factors
  • 88. Dyslipidemia in NAFD  The available evidence suggests that the risk for serious liver injury from statins is quite rare and patients with NAFLD and dyslipidemia are not at an increased risk for statin-induced hepatotoxicity. Hence, statins could be used by physicians to treat dyslipidemia in patients with NAFLD.  There are few data to suggest usage of statins as a treatment option for NAFLD. However, it continues to be a matter of debate and treatment of NAFLD with statins cannot be recommended at present. NAFD: Non-alcoholic fatty liver disease
  • 89. Dyslipidemia in NAFD  The available evidence suggests that the risk for serious liver injury from statins is quite rare and patients with NAFLD and dyslipidemia are not at an increased risk for statin-induced hepatotoxicity. Hence, statins could be used by physicians to treat dyslipidemia in patients with NAFLD.  There are few data to suggest usage of statins as a treatment option for NAFLD. However, it continues to be a matter of debate and treatment of NAFLD with statins cannot be recommended at present. NAFD: Non-alcoholic fatty liver disease
  • 90. Dyslipidemia in Cerebrovascular Disease  Acute ischemic stroke: Starting a statin is probably beneficial. Continue statin if patient is already receiving it. Statins are not contraindicated if patient is thrombolysis. It may be given by nasogastric tube in patients with dysphagia.  Acute ICH: Do not initiate statin if patient is not already on it. Do not discontinue statin if patient is already on a statin. Optimal blood pressure control is essential in those already on a statin.  Primary prevention of stroke: Statin treatment is recommended in adults with diabetes, or CAD, to lower the risk of a first stroke.  Secondary prevention of stroke: Statins are beneficial in preventing a second ischemic stroke, but do not prevent fatal stroke.  Patients with previous haemorrhagic stroke: Statins are best avoided in patients with previous lobar haemorrhage, but may be beneficial in preventing stroke in patients with previous basal-ganglionic haemorrhage who have a high risk for ischemic events. ICH: Intracerebral Hemorrahge
  • 91.  All patients with dyslipidemia should be screened for thyroid dysfunction.  Patients with dyslipidemia and overt hypothyroidism should be treated initially with thyroxine.  In patients with underlying risk for CAD, start low dose thyroxine and titrate upwards slowly. Assess CVD risk and start lipid-lowering drugs as per LAI recommendation for primary prevention of CVD.  Patients without underlying risk for CAD: wait for the patient to become euthyroid.  Statins or fibrates alone or in combination in patients with dyslipidemia and uncontrolled hypothyroidism carry a higher risk of myopathy.  Thyroid replacement in patients with sub-clinical hypothyroidism if serum TSH is >10 mIU/L, patients have high initial cholesterol levels or are elderly, are smokers or are positive for anti-thyroid peroxidase antibodies.  In patients with established vascular disease requiring thyroxine and lipid- lowering therapy, caution should be exercised with thyroxine dose to prevent onset of ischemic symptoms. Dyslipidemia in Thyroid Disorder CAD: coronary artery disease, CVD: Cardiovascular disease, TSH: Thyroid stimulating hormone
  • 92.  CVD risk assessment for IJD patients should be carried out [including total cholesterol and HDL-C (fasting or non-fasting)] during the stable disease state once every 5 years.  The use of statins and CVD risk management should follow the same principles as discussed in the Part1 of this consensus document. Management of ASCVD risk in inflammatory joint diseases (IJD) CVD: Cardiovascular disease, HDL-C: High density lipoprotein
  • 93. Familial Hypercholeterolemia (FH)  Lipid profile estimation of children to be done at 2 years of age in those with family history of FH and premature ASCVD.  LAI recommends the Simon Broome criteria for the diagnosis of FH.  In an established case of FH, LAI recommends estimation of Lp(a) levels.  Genetic testing and cascade screening should be performed wherever feasible.  Look for other ASCVD risk factors and manage them appropriately.  Strict dietary recommendations and lifestyle modifications as advised.  Drug therapy to be started at age 8 years or earlier in individualised cases.  LDL-C targets to be achieved:<70 mg/dL for HoFH and <100 mg/dL for HeFH in children and in adults <50 mg/dL in HoFH and 70 mg/dL in HeFH or at least 50% reduction in LDL-C from the baseline. HeFH: Heterozygous FH; HoFH: Homozygous FH; HF: Heart Failure, Lp(a): Lipoprotein, LDL-C: Low-denisty lipoprotein
  • 94. Inherited Hypertriglyceridaemia  All subjects should be screened for hypertriglyceridaemia with a fasting lipid profile. A non-fasting lipid profile may be performed as an initial step, but fasting TG estimation will be needed if TG is found to be high in the non-fasting sample  Exclude and treat secondary causes  Aggressive therapeutic lifestyle changes should be implemented  Subjects with TG > 200 mg/dL and <500 mg/dL – treat with a statin  Subjects with TG ≥ 500 mg/dL-treat with fibrates to prevent acute pancreatitis and later add statin to achieve non-HDL-C goal  Preferred drugs: Fibrates and high dose omega fatty acids; the role of niacin is controversial. TG; Triglycerides, HDL-C: High-density lipoprotein
  • 95. Lipids targets in HIV patients as recommended by LAI Dyslipidemia in HIV Effect of different statins in HIV patients with dyslipidemia LDL-C: Low-denisty lipoprotein, HDL-C: High density
  • 96.  Lifestyle modifications with special emphasis on smoking cessation, weight reduction and calorie restriction are important.  The recommendations for the evaluation and treatment of dyslipidemia as suggested by LAI regarding target goals for lipids should be strictly followed to help guide therapy.  At least, evidence available in the Indian population suggests the same. Dyslipidemia in HIV HIV: Human Immunodeficiency Virus, LDL-C: Low-density lipoprotein
  • 97.  Different types of statins are available to lower plasma lipids to guideline levels in patients with HIV, but they differ in their pharmacokinetic properties and drug interaction profiles. Simvastatin and lovastatin are contraindicated in patients taking PIs.  The other statins atorvastatin and rosuvastatin (apart from simvastatin and lovastatin) have better therapeutic effect in lowering LDL-C in HIV dyslipidemia. The addition of ezetimibe is another option. Fenofibrate and fish oil can be used in statin- unresponsive HIV dyslipidemia. Dyslipidemia in HIV HIV: Human Immunodeficiency Virus, LDL-C: Low-density lipoprotein
  • 98. Dyslipidemia in Women  Women with collagen vascular disorders, PCOS, preeclampsia, pregnancy-induced hypertension and gestational diabetes are at higher ASCVD risk.  The dyslipidemia management is similar in both men and women. However, in elderly frail women, treatment should be started with low dose statin in view of higher incidence of muscle-related adverse effects except in the very high risk group.  Statins remain the first-line cholesterol-lowering drug therapy for primary and secondary prevention in women. Statin dosage should be increased to the maximally tolerated dose before adding a non-statin drug if goal levels of LDL-C and non-HDL-C are not achieved.  Non-statin drugs may be considered as a primary drug for women with contraindications for, or intolerance to, statin therapy. PCOS: Polycystic ovarian syndrome, LDL-C: Low-denisty lipoprotein, HDL-C: High density lipoprotein
  • 99. Dyslipidemia in Elderly  CVD is the most common cause of death in the elderly. As in the young, elderly patients with dyslipidemia have an increased risk for CVD. • Secondary causes of dyslipidemia such as hypothyroidism, diabetes, CKD and drug effects should be considered in elderly patients.  The RRR of lipid-lowering therapy in elderly patients is similar to that in younger patients; however, the absolute benefit is higher than in younger patients.  Reductions in events with statin therapy are apparent within a few weeks to months even in older patients. RRR: Relative Risk Reduction, CKD: Chronic kidney disease, CVD: Cardiovascular
  • 100. Dyslipidemia in Elderly  Side effects of lipid-lowering therapy are similar in the old and young. However, in very elderly frail patients, lower doses may be appropriate.  Secondary prevention: Statin therapy should be instituted in all appropriate elderly patients as above with treatment goals similar to younger patients. Dose of statins may need to be individualized in the elderly based on frailty and other comorbidities.  Primary prevention: Elderly subjects with a reasonable life expectancy should be treated similar to younger patients as per risk stratification algorithm proposed in the previously published LAI expert consensus statement RRR: Relative Risk Reduction, CKD: Chronic kidney disease, CVD: Cardiovascular
  • 101. Post-Heart Transplantation  Baseline lipid levels should be obtained for all patients after heart transplantation.  The strict control of modifiable risk factors including hypertension, diabetes, dyslipidemia, smoking and obesity should be reinforced.  In adults, the use of statins beginning 1 to 2 weeks after heart transplantation is recommended regardless of cholesterol levels. CAV: Cardiac Allograft Vasculopathy, LDL-C: Low-denisty
  • 102. Post-Heart Transplantation  Pravastatin shows the least interaction with cyclosporine to produce myopathy, making it the drug of choice.  Rosuvastatin up to 10 mg/day may be given as a second choice.  Addition of a statin to a cyclosporine–sirolimus regimen produces multiple beneficial effects like reduced cholesterol levels, decreased acute rejection episodes, decreased incidence of CAV and improved survival.  In heart transplant recipients, a strategy of lowest achievable LDL-C levels with maximally tolerated dose of statin by slowly up-titrating the statin dose is justified. CAV: Cardiac Allograft Vasculopathy, LDL-C: Low-denisty
  • 103. Sources • Stone Nj, Robinson J, Lichtenstein Ah, Bairey Merz Cn, Lioyd-jones Dm, Blum Cb, Mcbride P, eckel Rh, Schwartz Js, Goldberg Ac, Shero St, Gordon D, Smith Sc Jr, Levy D, Watson K, Wilson Pw. 2013 ACC/AHA Guideline On The Treatment Of Blood Cholesterol To Reduce Atherosclerotic Cardiovascular Risk In Adults: A Report Of The American College Of Cardiology/American Heart Association Task Force On Practice Guidelines. J Am Coll Cardiol. 2013 Nov 7. Pii: S0735-1097 • John F. Keaney, Jr., M.D., Gregory D. Curfman, M.D., And John A. Jarcho, M.D. A Pragmatic View Of The New Cholesterol Treatment Guidelines. N Engl J Med 2014; 370:275-278